Você está na página 1de 39

Perguntas de Quizzing Conectadas

Capítulo 2

Alberto era dono da Blackacre, que era vizinha da Whiteacre, de propriedade de Berta.
Em 1998, Alberto construiu sua garagem sem fazer uma vistoria e, como resultado,
parte da nova garagem se estendeu cerca de dois metros sobre a linha da propriedade
em Whiteacre. Em 2005, Alberto morreu. Seu testamento deu Blackacre para sua filha
Cara. Ela se mudou e, em 2007, decidiu vender a Blackacre para Darrell, que
prontamente se mudou. Em 2012, Berta estava tentando vender Whiteacre para um
potencial comprador, que descobriu que a garagem estava acima da linha. Em 2012,
Berta processou Darrell para exigir que ele mudasse a garagem ou pagasse
indenizações. Darrell reivindica o título por usucapião, que exige dez anos de posse
nesta jurisdição. Qual das seguintes afirmações é verdadeira?

(A) Darrell não pode cumprir o período legal de posse, porque ele pode pegar a posse
de Cara mas não Alberto.

(B) Darrell não pode se agarrar à posse de Cara ou Alberto para cumprir o requisito
estatutário.

(C) Darrell deve ser capaz de cumprir o requisito de usucapião usando a posse
de Cara e Alberto além da sua.

(D) Darrell não pode alegar usucapião em hipótese alguma, pois não preenche o
requisito da adversidade em qualquer jurisdição.
Lógica:
A resposta correta é C. Darrell está na posse há apenas cinco anos, portanto, para cumprir o
período legal de dez anos, Darrell precisará lidar com a posse de Alberto e Cara. Um usucapião
pode "atacar" a posse de possuidores anteriores se eles estiverem em privação, o que significa
apenas que há alguma conexão razoável entre eles que torna justo dar ao possuidor atual crédito
pela posse anterior. Normalmente, alguém que herda ou é inventado um imóvel em testamento,
como Cara, tem essa conexão e o mesmo acontece com um comprador de boa-fé do imóvel, como
Darrell. Assim, as respostas A e B estão incorretas. O combate não seria permitido apenas quando
não houvesse conexão entre um possuidor e outro. Portanto, não parece haver qualquer razão para
que um tribunal negue a Darrell o direito de abordar esses períodos anteriores de posse. D está
incorreto porque Darrell não tem permissão para invadir a propriedade de Berta; portanto, ele é
adverso. Em algumas jurisdições, o fato de Darrell não saber que estava invadindo pode destruir
sua adversidade, mas a maioria das jurisdições consideraria a posse sem permissão como adversa.
Capítulo 3

1.
O concedeu Blackacre "à Universidade enquanto o tabaco não for consumido de qualquer forma
no local, e se o tabaco for consumido, Blackacre reverterá para O, seus herdeiros, sucessores e
cessionários". Qual das seguintes afirmações é verdadeira?
(A) A universidade tem uma taxa simples sujeita a uma condição posterior.

(B) Universidade tem uma taxa simples determinável.

(C) A universidade tem um contingente remanescente.

(D) O interesse da Universidade é nulo nos termos da Regra Contra as Perpetuidades.

Lógica:
A resposta correta é B. A universidade tem um interesse presente, então a resposta C está
incorreta porque um restante é um interesse futuro. Como o interesse da Universidade pode durar
para sempre, é uma taxa simples. No entanto, como pode terminar mais cedo, é uma forma de
taxa inviável. Há duas possibilidades: taxa simples sujeita a uma condição posterior e taxa
simples determinável. Um patrimônio determinável tem uma duração (por exemplo, "enquanto
houver" ou "até") embutido na concessão, enquanto um patrimônio condicional tem uma
condição anexada após uma concessão absoluta ("desde que, no entanto" ou "na condição de
que"). Além disso, uma taxa determinável é normalmente seguida por uma reversão automática
em caso de violação, enquanto a taxa condicional tem uma rescisão opcional ("O pode retomar a
posse"). A concessão, nesse caso, é duradoura e a rescisão automática. Portanto, a Universidade
tem uma taxa simples determinável, e a resposta A está incorreta. A resposta D está incorreta,
pois o interesse da Universidade não é nulo sob a Regra porque a Regra se aplica apenas a
interesses futuros (e, a propósito, o interesse de O também não é afetado porque os interesses
retidos pelo concedente não estão sujeitos à Regra).

2.

Em 16 de dezembro de 2018, o Condado de Cook transmitiu muito "para a cidade de Chicago


pela vida de George Lucas, restante para o Lucas Museum of Narrative Art, mas se esse lote não
for usado para uma exposição permanente da Estrela da Morte original, então o mesmo reverterá
para proprietário".
Que interesse adquiriu o Museu?

(A) Taxa simples determinável

(B) Taxa simples sujeita a condição posterior

(C) Taxa simples absoluta


(D) Nenhuma das situações acima

Lógica:
A resposta correta é (D). O Museu adquiriu um remanescente sujeito a desinvestimento, um
interesse futuro. (A), (B) e (C) estão incorretos porque cada um nomeia um interesse possessório
presente. O único interesse possessório presente nesta concessão é a propriedade vitalícia que se
encontra na cidade de Chicago para a vida de Lucas.

Capítulo 5

Após o casamento em 2010, Juan Carlos e Jamie compraram uma casa, assumindo o
título de inquilinos por inteiro. Jamie agora acha que a casa não atende mais às
necessidades da família, e ela quer se mudar. Juan Carlos ama a casa e não tem
interesse em se mudar. Jamie procurou conselhos de um amigo, que sugeriu que
Jamie entrasse com uma ação de partilha para forçar a venda da casa.
Jamie terá sucesso?

(A) Sim, porque qualquer titular de juros pode lograr êxito em um pedido de partilha.

(B) Sim, porque Juan Carlos se recusa a vender.

(C) Não, porque o arrendamento não pode ser cortado.

(D) Não, porque um arrendamento por inteiro pode ser cortado.


Lógica:
A resposta correta é (C). O arrendamento por inteiro, disponível apenas para casais, não pode ser
cortado unilateralmente através de uma acção de partilha. Assim, (A) e (D) estão incorretos. (B)
também está incorreta, pois a recusa de venda de Juan Carlos não tem qualquer relação com a
restrição à separação unilateral de bens detidos pela totalidade.
Leitura recomendada:
Páginas 245-266 de Dukeminier, Propriedade, Edição Concisa 2E
2

Andy, Barb e Chet herdaram Blackacre, uma fazenda de 120 acres, em partes iguais
como inquilinos em comum. Pouco tempo depois, Andy vendeu sua participação de um
terço para Malls, Inc., que quer todo o terreno para que possa construir um novo
shopping. Barb não quer que a terra seja dividida, particularmente não por venda,
porque ela ama a terra. A Malls, Inc., pede ao tribunal a partilha por venda, alegando
que dividir o terreno em três partes o tornará menos valioso e que seria difícil fazer com
que as ações saíssem uniformemente. Qual das seguintes afirmações é verdadeira na
maioria das jurisdições?
(A) Barb pode anular a venda da parte de Andy para Malls, Inc., porque Andy não tinha
sua permissão.

(B) A Malls, Inc., só pode pedir a partilha se puder demonstrar que permanecer em um
coarrendamento é impossível ou impraticável.

(C) Um tribunal ordenará uma venda, a menos que Barb possa provar que a partilha
em espécie será mais benéfica para todas as partes.

(D) A Malls, Inc., pode obter a partilha por venda se puder demonstrar que a
partilha em espécie é inviável ou resultará em grande prejuízo para as partes.

Lógica:
A resposta correta é D. A resposta A está incorreta porque qualquer coinquilino pode alienar
(vender ou doar) sua parte do espólio a qualquer momento, sem permissão, a menos que haja
algum tipo de restrição à alienação de seu interesse. (Se houvesse tal contenção, teria que ser
razoável para ser válido, como que A, B e C não poderiam vender Blackacre durante suas vidas.)
Aqui não há restrição, então Andy tem o direito de alienar sem permissão. Após a venda, a Malls,
Inc., torna-se inquilina em comum com Barb e Chet. A resposta B está incorreta porque qualquer
coinquilino tem o direito de dividir a qualquer momento (novamente, desde que não haja restrição
ao seu interesse, o que não há aqui). Um coinquilino não precisa mostrar nenhum motivo para
buscar a partição. Então, a única questão real aqui é se a partição será em espécie (física) ou por
venda. Normalmente, os tribunais favorecem a partilha em espécie porque uma venda irá
desapropriar as partes de seus bens. Assim, o teste usual exige que uma parte que busca a partilha
por venda prove que a partilha em espécie é inviável ou resultará em grande prejuízo para as partes
(por exemplo, se reduzirá muito o valor do terreno a ser quebrado em pedaços menores). Portanto,
a resposta C está incorreta porque Barb não tem o ônus de provar que a partilha em espécie é mais
benéfica. Em vez disso, a divisão em espécie será usada, a menos que a Malls, Inc., prove que
será inviável ou resultará em grande prejuízo. Portanto, a Resposta D é a melhor resposta.
Leitura recomendada:
Páginas 254-262 de Dukeminier, Propriedade, Edição Concisa 2E

3.
Ernst, Frieda e Gertrude compraram Blackacre, uma casa de três quartos, e tomaram o
título de inquilinos em comum. Eles compraram a Blackacre como investimento e
planejavam alugá-la. No entanto, Frieda e Gertrudes adoraram a casa e decidiram se
mudar. Ernst quer que Frieda e Gertrudes lhe paguem o aluguel por morar em uma
casa que é um terço dele, afinal. Frieda e Gertrudes recusam-se a fazê-lo e dizem-lhe
que ele é livre para ir morar com eles se quiser. Qual das seguintes afirmações é
verdadeira?

(A) Frieda e Gertrudes devem a Ernst dois terços do valor do aluguel da casa.

(B) Frieda e Gertrudes devem a Ernst um terço do valor do aluguel da casa.


(C) Frieda e Gertrude não devem aluguel a Ernst, mas o valor pode ser levado em
conta na determinação da responsabilidade por carregar encargos como seguros
em uma ação contábil.

(D) Se os três fossem inquilinos comuns em vez de inquilinos em comum, Frieda e


Gertrudes teriam que pagar aluguel a Ernst.
Lógica:
A resposta correta é C. Qualquer coinquilino pode possuir a totalidade do imóvel sem
responsabilidade pelo aluguel para o outro coinquilino. Cada condômino tem o direito de possuir o
imóvel, por isso a teoria é que não há responsabilidade, ainda que impeça que o imóvel seja
utilizado para renda produtiva. A única exceção é a demissão, quando um coinquilino é impedido de
exercer seu direito à posse pelos outros coinquilinos. Neste caso, Frieda e Gertrude convidaram
explicitamente Ernst para compartilhar a posse com eles, então não há nenhum caso para a
demissão. Portanto, as respostas A e B estão incorretas. Se a expulsão tivesse ocorrido, Ernst
receberia um terço do valor do aluguel, e não dois terços. A resposta D está incorreta porque não
há diferença entre um arrendamento conjunto e um arrendamento comum no que diz respeito aos
direitos e responsabilidades dos coarrendatários. A resposta C está correta, pois embora Ernst não
possa exigir que os coinquilinos na posse paguem aluguel, o valor de sua ocupação pode ser
levado em conta na determinação da responsabilidade por carregar encargos. Como eles estão
recebendo o valor da casa, faz sentido que paguem pelo seguro, por exemplo.
Leitura recomendada:
Páginas 262-266 de Dukeminier, Propriedade, Edição Concisa 2E

4.
Gramps queria dar Blackacre para seus dois netos, Nick e Nora. Na segunda-feira, ele
deu a Nick uma escritura para Blackacre que dizia: "Gramps transmite a Nick um meio
interesse em Blackacre". Na terça-feira, ele deu uma escritura a Nora, que afirmava
"Gramps transmite um meio interesse em Blackacre para Nora, em locação conjunta
com Nick". Nick morreu pouco depois, deixando um testamento dando todos os seus
bens para sua esposa, Wanda. Qual das seguintes afirmações é verdadeira?

(A) Wanda não leva nada, por causa do direito de sobrevivência de Nora.

(B) Wanda tem um meio interesse em Blackacre como inquilino em comum com
Nora.

(C) Wanda não leva nada, porque Gramps não pode transmitir Blackacre dessa
maneira.

(D) Wanda não leva nada, porque Gramps pretendia que apenas seus netos fossem
beneficiados.
Lógica:
A resposta correta é B. Gramps pode ter a intenção de criar um arrendamento conjunto com Nick e
Nora, mas ele falhou. Ele não transmitiu a Nick e Nora ao mesmo tempo com o mesmo instrumento,
falhando assim duas das unidades de título exigidas. Além disso, sua transmissão para Nick nem
menciona um arrendamento conjunto. Então, quando ele transmitiu um meio interesse para Nick, o
resultado foi um arrendamento em comum entre Gramps e Nick. Ele então transmitiu seu meio
interesse restante para Nora, criando um arrendamento em comum entre Nick e Nora, apesar da
linguagem da transmissão. Quando Nick morreu, então, seu interesse iria para Wanda sob seu
testamento, resultando em um arrendamento em comum entre Nora e Wanda. A resposta A está,
portanto, incorreta; não há direito de sobrevivência em um arrendamento em comum. C está
incorreto porque Gramps conseguiu transmitir seus interesses, apenas não em locação conjunta. D
não pode estar certo; mesmo que o vovô pretendesse que apenas seus netos se beneficiassem, ele
não limitava sua transmissão a uma propriedade vitalícia ou de qualquer outra forma. Portanto,
cada um deles tinha metade dos juros em honorários simples absolutos, que são debocháveis,
hereditários e alienáveis. Certamente, na common law, ele deveria ter dito "para Nick e seus
herdeiros", mas essa linguagem não é mais necessária e presumimos que Nick tinha a intenção de
ter sua metade do interesse em honorários simples absolutos.
Leitura recomendada:
Páginas 245-247 de Dukeminier, Propriedade, Edição Concisa 2E

5.
Em 1990, H comprou Blackacre, um rancho de 300 acres no Tennessee, e viveu nele
por muitos anos. Em 2010, ele se casou com W, mas não lhe transmitiu nenhum
interesse em Blackacre. O casal morava em um apartamento alugado na cidade,
enquanto o rancho era usado para criar cavalos. Vários anos depois, o casal decidiu se
aposentar no Texas. Eles compraram um trailer em Houston e moraram nele. O Texas
é um estado de propriedade comunitária, enquanto o Tennessee não é. Logo depois, H
vendeu a Blackacre e colocou os lucros (US$ 1 milhão) em uma conta de ações
apenas em seu nome. H então morreu, domiciliado no Texas. Seu testamento deixou
todos os seus bens para D, sua filha de um casamento anterior. O trailer valia US$ 200
mil e tinha o título em nome de H. O Tennessee tem um estatuto de partilha eletivo,
dando aos cônjuges sobreviventes um terço de participação no patrimônio de um
cônjuge falecido, enquanto o Texas não. Desconsiderando as questões de propriedade
e dower, qual das seguintes é verdadeira?

(A) Quando H e W se mudaram para o Texas, W automaticamente passou a ter direito


a uma metade da participação na fazenda Blackacre como propriedade comunitária e,
portanto, deveria receber metade da conta de ações.

(B) Em razão da comunhão de bens, W era coarrendatário da RV, apesar da forma


como esta era titulada, sendo, portanto, a única proprietária da mesma aquando da
morte de H.

(C) W poderia usar a provisão de ações eletivas do Tennessee para obter uma parte de
um terço de todos os ativos do patrimônio de H, apesar da vontade.

(D) W possui metade do RV, mas nenhum da conta de ações.

Lógica:
A resposta correta é D. Esse é o problema dos "casais migrantes". Quando o casal estava
domiciliado no Tennessee, W estava protegido pela cláusula de partilha eletiva do estado. Se H
tivesse morrido quando eles moravam lá, ela teria direito a um terço da Blackacre. Quando o casal
se mudou para o Texas, no entanto, ela perdeu essa proteção. Se Blackacre não tivesse sido
vendido, a resposta provavelmente seria diferente, porque a lei do local da propriedade real (neste
caso Tennessee) normalmente rege a disposição de bens imóveis na morte. Quanto à propriedade
restante, a lei do Texas regerá a propriedade porque é onde H estava domiciliado no momento da
morte. A conta de ações não pode ser considerada patrimônio comum, pois não provém dos
rendimentos de nenhum dos cônjuges durante o casamento. H possuía-o antes do casamento.
Portanto, a conta de estoque também é propriedade separada. O RV, no entanto, foi utilizado no
casamento e deve ser considerado patrimônio comum, independentemente de como é titulado. A
resposta A está, portanto, incorreta; A propriedade não é convertida automaticamente em
propriedade comunitária quando um casal se muda para um estado de propriedade comunitária. A
resposta B está errada porque a propriedade comum não faz de W um condômino do RV; Isso lhe
daria apenas meio interesse nisso. O casal poderia tê-lo titulado em locação conjunta, mas não o
fizeram. A resposta C também está errada. A provisão de ações eletivas do Tennessee não se
aplica porque H morreu domiciliado no Texas.
Leitura recomendada:
Páginas 266-267 de Dukeminier, Propriedade, Edição Concisa 2E

6.
Kim e Kanye eram casados e donos de uma casa de férias chamada Greenacre como
inquilinos. Uma noite, Kim bebeu demais e sofreu um acidente. Ela bateu em um carro
dirigido por Talia, ferindo-a gravemente. Talia processou e obteve uma sentença de
US$ 2.000.000 contra Kim. Como Kim não tinha seguro suficiente para pagar esse
valor, Talia quer satisfazer a sentença contra Greenacre. Qual das seguintes
afirmações é verdadeira na maioria das jurisdições americanas que permitem o
arrendamento por inteiro?

(A) Talia deve ser capaz de alcançar a metade do interesse de Kim na Greenacre,
porque as sentenças de responsabilidade civil são geralmente uma exceção à regra
contra a rescisão unilateral.

(B) Talia não poderá penhorar Greenacre, pois os bens mantidos em


arrendamento pela totalidade não estão sujeitos a dívidas de apenas um dos
cônjuges.

(C) Talia não poderá chegar a Greenacre agora, mas pode criar uma penhora sobre o
imóvel, que ela pode executar na venda do imóvel.

(D) Talia deveria ser capaz de alcançar a totalidade de Greenacre, porque Kim tinha
direito à posse do todo.
Lógica:
A resposta correta é B. Os bens detidos em arrendamento pela totalidade não podem ser
alcançados apenas pelos credores de um dos cônjuges. Ver Sawada v. Endo ou RBS Citizens v.
Ouhrabka. A maioria das jurisdições não abre exceção para sentenças de responsabilidade civil,
portanto, A está incorreta. A resposta C não é a regra na maioria das jurisdições que usam TBE,
com exceção das obrigações fiscais federais. Kim certamente tinha direito à posse do todo, como
afirma a resposta D, mas toda a ideia de locação pela totalidade é evitar que um dos cônjuges
onere unilateralmente o bem, de modo que o direito à posse não é relevante.
Capítulo 6

Kaya aluga um apartamento loft no centro de Arcadia City para o estudante de direito
Cliff por um período de três anos a um aluguel mensal de US $ 2.500. Um ano depois,
depois de receber um estágio fora do estado, Cliff "subaluga, transfere e cede" a
propriedade a seu colega Thomas por "um período de um ano a partir da data de hoje".
Depois disso, nem Thomas nem Cliff pagam aluguel para Kaya. Kaya tem algum direito
contra Cliff?

(A) Sim, Kaya tem direitos contra Cliff porque a transferência para Thomas é uma
cessão, e Thomas não pagou aluguel.

(B) Sim, Kaya tem direitos contra Cliff porque a transferência para Thomas é uma
sublocação, e Thomas não pagou aluguel.

(C) Não, Kaya não tem direitos contra Cliff porque a transferência para Thomas é uma
cessão, e Thomas é responsável perante Kaya pelo aluguel total.

(D) Não, Kaya não tem direitos contra Cliff porque a transferência para Thomas é uma
sublocação, e Thomas é responsável por Kaya pelo aluguel total.
Lógica:
A resposta correta é (B). Cliff tem um contrato de três anos e transferiu apenas um ano para
Thomas. Dado que ele transferiu menos do que seu interesse total - note que Cliff ainda tem um
ano restante em seu contrato de locação depois que Thomas vacina - essa transferência é
caracterizada como um subarrendamento. Como o locador e o inquilino original permanecem tanto
na privação da propriedade quanto na privação do contrato, o inquilino permanece responsável pelo
aluguel. Portanto, (A) e (C) estão incorretos. (D) também é incorreta porque confunde qual das
partes de um contrato de sublocação é legalmente responsável perante o locador pelo aluguel. A
parte que cobra o aluguel em uma sublocação é sempre o inquilino original (o sublocador), não o
sublocatário. Ver Ernst v. Conditt, 390 S.W.2d 703 (Tenn. 1964).
Leitura recomendada:
Páginas 294-308 de Dukeminier, Propriedade, Edição Concisa 2E
2

Lance era dono de um prédio de apartamentos. Tracy queria alugar o Apartamento 2


de Lance, mas ela notou que havia vários problemas com ele. A pia estava entupida,
havia alguns fios expostos onde uma luminária estava se soltando, e havia um odor
severo de mofo vindo do armário, onde o teto parecia mostrar sinais de danos
persistentes à água. Lance reconheceu os problemas, mas disse que não tinha tempo
ou dinheiro para consertar o apartamento. "Eu vou te dizer o quê", Lance disse a Tracy,
"normalmente este lugar sai por US $ 600 por mês, mas dada a sua condição, posso
deixá-lo por US $ 400. Mas o contrato de locação dirá que você entende que não há
garantia em relação à condição." Tracy assinou o contrato, mas vários meses depois,
ela parou de pagar o aluguel, alegando que os odores de mofo eram avassaladores e
os outros defeitos tornavam o local inseguro e insalubre. Qual das seguintes
afirmações é verdadeira?

(A) Tracy não pode alegar a violação da garantia implícita de habitabilidade, porque ela
não se mudou e, portanto, não há despejo construtivo.

(B) Tracy não pode alegar uma violação da garantia implícita de habitabilidade, porque
ela a renunciou no contrato de locação em troca de um preço de aluguel mais baixo.

(C) Tracy pode alegar uma violação da garantia implícita de habitabilidade, mas apenas
se alguns dos defeitos reclamados violarem o código de habitação.

(D) Tracy tem um bom argumento para uma violação da garantia implícita de
habitabilidade, que pode ser usada em defesa se Lance entrar com uma ação por
aluguel não pago.

Lógica:
A resposta correta é D. A garantia implícita de habitabilidade (IWH) protege os inquilinos
residenciais de condições que afetam significativamente a segurança e a saúde. A resposta A não
está correta, pois, ao contrário do pacto de gozo tranquilo, o IWH não exige que o inquilino se mude
e pleiteie um despejo construtivo. A resposta B também está incorreta porque os tribunais não
permitem que o IWH seja dispensado, mesmo quando o inquilino recebe um aluguel mais baixo em
troca de uma tentativa de renúncia. Algumas leis de locação residencial permitem isenções em
circunstâncias limitadas, como casas unifamiliares, mas não em um caso como este. A resposta C é
muito limitada; na maioria das jurisdições, mesmo que as violações não estejam cobertas no código
de habitação, desde que as condições tornem as instalações inseguras ou insalubres, o IWH é
violado. Nesse caso, o mofo, os fios expostos e a pia entupida parecem suficientes para justificar
que o IWH foi violado.
Leitura recomendada:
Páginas 334-344 de Dukeminier, Propriedade, Edição Concisa 2E
3

Lana arrendou Blackacre para Tad por um período de dez anos, de 1º de janeiro de
2006 a 31 de dezembro de 2015, por US$ 5.000 por mês. Em 2008, Tad entrou em um
acordo com Fiona para transferir esse arrendamento para ela pelo resto do período de
aluguel. Fiona concordou em assumir todas as responsabilidades do aluguel e
concordou em pagar o aluguel mensal de US $ 5.000 diretamente para Lana. Lana
aprovou o acordo, mas não aderiu ao contrato. Dois anos depois, em 2010, Fiona
transferiu o mandato restante para o Sal. Novamente, Lana aprovou o acordo e Sal
concordou em assumir todas as responsabilidades do contrato e pagar o aluguel
mensal de US$ 5.000 diretamente para Lana. Um ano depois, em 2011, Sal deixou de
pagar a renda e Lana quer entrar com uma ação para a renda devida. Qual dessas três
partes ela pode entrar com ação?

a) Tad e Sal
(B) Tad, Fiona e Sal

(C) Apenas Sal

(D) Tad apenas


Lógica:
A resposta correta é A. As duas transferências para Fiona e Sal são cessões porque são para todo
o prazo restante e transferem todos os direitos e responsabilidades sob o contrato de locação
principal. Portanto, quando Fiona assume como cessionária, Lana está em privação de contrato
ainda com Tad. (Ela sempre estará em privação de contrato com Tad a menos que ela o liberte das
obrigações contratuais.) C está, portanto, incorreto. Lana está então em privação de propriedade
com Fiona porque Fiona assumiu a propriedade de Tad. Quando Fiona transfere a propriedade para
Sal, outra cessão, Sal fica em privação de propriedade com Lana e D está, portanto, incorreto.
Fiona não está mais em privação de propriedade naquele momento e nunca esteve em privação de
contrato. Isso torna B incorreto. Lana pode processar Tad sob prividade de contrato e Sal sob
prividade de propriedade.
Leitura recomendada:
Páginas 294-300 de Dukeminier, Propriedade, Edição Concisa 2E
4

Sorte que o senhorio e Tessa, a inquilina, entram em um contrato de locação mês a


mês em 1º de novembro de 2018 que especifica um aluguel de US $ 1.200 por mês.
Tessa sempre pagou o aluguel em dia e integralmente, mas agora está considerando
mudar de casa com os pais para economizar. A partir da data de hoje (15 de
dezembro), a data mais próxima após a qual Tessa não será mais responsável pelo
aluguel seria:

(A) 31 de dezembro.

(B) 31 de janeiro.

(C) 1º de janeiro.

(D) 28 de fevereiro.
Lógica:
A resposta correta é (B). Uma locação periódica é executada por um determinado período de
tempo, neste caso mês a mês. Não há fatos presentes que sugiram que esse acordo criou um
contrato de locação periódica ano a ano. Em um contrato de locação periódica mensal, qualquer
uma das partes pode rescindir o contrato mediante aviso prévio com pelo menos um mês de
antecedência. Aqui, Tessa decide se mudar em 15 de dezembro. Ela precisaria dar pelo menos um
mês de aviso prévio (pelo menos 30 dias) para evitar a responsabilidade pelo aluguel. Como
resultado, (A) e (C) devem estar incorretos, pois essas datas forneceriam apenas 16 ou 17 dias,
respectivamente, de aviso prévio. (D) também deve estar incorreta, pois não há regra que exija que
seja dado aviso prévio de dois meses (aqui, um período igual a 2x a duração do contrato de
locação) para rescindir um contrato de locação periódica mês a mês.
Leitura recomendada:
Páginas 281-287 de Dukeminier, Propriedade, Edição Concisa 2E
5
Lora era proprietária do Mayfair Mall e alugou um dos edifícios para a McBurgers
Restaurants por um período de cinco anos. Vários meses após a locação, McBurgers
teve problemas para fazer com que o ralo do piso funcionasse corretamente, o que
resultou em água no chão sempre que os funcionários colocavam mais de um galão de
água na pia. O contrato de arrendamento indica que a McBurgers tomou as instalações
"tal como estão" e inclui uma disposição que impõe à McBurgers o dever de "fazer
todas as reparações necessárias nas instalações". Qual das seguintes opções é
verdadeira na maioria das jurisdições?

(A) A garantia implícita de habitabilidade provavelmente foi violada aqui, porque esse
problema afeta a saúde e a segurança e não pode ser dispensado na locação.

(B) A McBurgers não poderá alegar uma violação da garantia implícita de


habitabilidade, porque não sofreu "despejo construtivo".

(C) McBurgers não poderá alegar uma violação do pacto de gozo silencioso, porque
esse problema não tem nada a ver com ruído.

(D) McBurgers teria uma reivindicação mais forte por uma violação do pacto de
gozo tranquilo se a causa do problema de drenagem for rastreada a uma área
comum no Mayfair Mall fora do restaurante.

Lógica:
D é a resposta correta. Lembre-se de que, na maioria das jurisdições, a garantia implícita de
habitabilidade se aplica apenas a residências, portanto, as respostas A e B podem ser descartadas.
B também é falso porque o IWH não exige um despejo construtivo. A lei geralmente dá aos
contratos de locação comercial muito mais espaço para negociar deveres de reparação, dada a
variedade de situações apresentadas e o poder de barganha relativamente igual. O pacto de fruição
tranquila aplica-se aos estabelecimentos comerciais. No entanto, a resposta C está incorreta porque
o gozo "silencioso" não se refere apenas ao ruído, mas sim a qualquer coisa que perturbe o uso e a
fruição do local pelo inquilino. A interferência deve decorrer, no entanto, de algo que o locador
causa ou tem o dever de remediar. Portanto, D está correto. Neste caso, a Lora não é responsável
pelo estado das instalações arrendadas nos termos do contrato de arrendamento. No entanto, os
proprietários são responsáveis pela manutenção e reparação das áreas comuns, por isso, se o
problema tiver origem fora das instalações arrendadas, a McBurgers deve ter um bom caso.
Leitura recomendada:
Páginas 334-344 de Dukeminier, Propriedade, Edição Concisa 2E
6

Em 1º de julho, o inquilino Tom alugou um apartamento do locador Lu mensalmente


por US$ 1.000/mês por um período mínimo de dez meses. As duas partes
concordaram com um aperto de mão e um acordo verbal em vez de escrever qualquer
coisa. Quando Tom se mudou, o apartamento estava em boa forma. Em 1º de
novembro, no entanto, o banheiro de Tom parou de funcionar e a pia da cozinha
entupiu. Tom ligou para Lu imediatamente e Lu disse que iria "cuidar disso". Três dias
depois, no entanto, nada havia sido feito, então Tom notificou Lu por escrito que queria
agir rapidamente sobre esses defeitos. Em resposta, Lu passou e mexeu no vaso
sanitário e na pia, mas não conseguiu consertá-los. Ele disse que precisaria "ligar para
meu amigo Brian. Ele pode consertar essas coisas." Tom não podia cozinhar porque
não podia usar a pia e tinha que usar o banheiro do vizinho ou descer para o saguão se
o vizinho não estivesse em casa. Em 1º de dezembro, Lu ainda não havia resolvido os
problemas, então Tom não pagou seu aluguel. Janeiro chegou e os problemas ainda
não foram resolvidos, então Tom também não pagou o aluguel de janeiro. Lu
finalmente resolveu o problema em 15 de janeiro e agora quer que Tom pague o
aluguel atrasado. Qual das doutrinas a seguir é a melhor defesa de Tom para essa
afirmação?

(A) Pacto de gozo tranquilo

(B) Garantia implícita de habitabilidade

(C) Reparação e dedução de recurso

(D) Estatuto das Fraudes


Lógica:
A resposta correta é B. A garantia implícita de habitabilidade exige que o senhorio mantenha as
instalações em condições adequadas e habitáveis. Uma violação ocorre se houver uma
interferência material nas condições habitáveis, o que parece ser. Além disso, o inquilino deve
notificar o locador do defeito e dar um prazo razoável para saná-lo, o que Tom fez aqui. Portanto, o
IWH seria uma defesa viável. A resposta A está incorreta porque o pacto do gozo tranquilo não se
aplica. Tom não saiu do apartamento (ou seja, não foi despejado construtivamente) e, portanto, não
pode usar essa doutrina. Tom poderia ter usado o remédio de reparo e dedução para essas
violações, mas isso exigiria que ele contratasse alguém para consertar os defeitos sozinho e, em
seguida, deduzir os custos de reparo, o que ele não fez. Assim, a resposta C está incorreta. O
Estatuto das Fraudes exige que apenas os contratos de arrendamento superiores a um ano sejam
por escrito para serem exequíveis. Portanto, o Estatuto das Fraudes não vai ajudar o Tom aqui.
(Você também pode observar que exceções, como o desempenho de peças, podem tornar a
locação exequível, mesmo que o Estatuto de Fraudes se aplique.) A resposta D está incorreta.
Leitura recomendada:
Páginas 334-344 de Dukeminier, Propriedade, Edição Concisa 2E
7

Margie's Muffins assinou um contrato de cinco anos para um espaço de padaria


comercial em Arcadia City, de propriedade da Langham Realty Co. Depois de quatro
anos, Margie decidiu se aposentar mais cedo e planejava sublocar o espaço para
Christine's Croissants, uma padeira concorrente na cidade. Margie pediu a seu
advogado que redigisse um contrato de sublocação de um ano para cobrir o acordo. O
acordo incluía uma cláusula que a Christine's pagava aluguel diretamente a Langham,
que "a Margie's Muffins está liberada de todas as obrigações sob o contrato de locação
original" e que "o sublocatário concorda em assumir todos os deveres e obrigações sob
o contrato de locação original". Margie's, Christine's e Langham assinaram o acordo.
Depois de seis meses no espaço, Christine deixa de pagar aluguel. Langham processa
Margie's por falta de pagamento de aluguel.
É provável que Langham seja bem-sucedido?

(A) Sim, pois o inquilino original é responsável pelo aluguel caso um sublocatário deixe
de pagar.

(B) Sim, porque o inquilino original permanece em contrato privado com o locador.

(C) Não, porque o senhorio consentiu no arranjo.

(D) Não, porque a inquilina originária cedeu os seus direitos sobre o espaço e já
não se encontra em privação de património ou de contrato privado com o
senhorio.

Lógica:
A resposta correta é (D). A análise gira em torno de se o contrato cria uma cessão ou uma
sublocação. Uma vez que Margie não manteve nenhum interesse reversivo na propriedade
("sublocação" é para o último ano do prazo original de cinco anos do contrato de locação de Margie)
e como o segundo acordo incluiu uma liberação de todas as obrigações para Margie, este acordo é
mais provável de ser considerado uma cessão. Ver Ernst v. Conditt. (A) está incorreta porque não
distingue entre uma sublocação e uma cessão. Em uma cessão, a privação de bens e o contrato
entre o inquilino original e o locador são destruídos. (B) está incorreto, pois o segundo contrato
continha linguagem que liberava o locatário original das avenças do contrato original. (C) está
incorreta porque o consentimento do locador não é dispositivo em questão de responsabilidade do
locatário.
Leitura recomendada:
Páginas 294-308 de Dukeminier, Propriedade, Edição Concisa 2E
8

Em 1º de julho, o inquilino Tom alugou um apartamento do locador Lu mensalmente


por US$ 1.000/mês por um período mínimo de dez meses. As duas partes
concordaram com um aperto de mão e um acordo verbal em vez de escrever qualquer
coisa. Quando Tom se mudou, o apartamento estava em boa forma. Em 1º de
novembro, no entanto, o banheiro de Tom parou de funcionar e a pia da cozinha
entupiu. Tom ligou para Lu imediatamente e Lu disse que iria "cuidar disso". Três dias
depois, no entanto, nada havia sido feito, então Tom notificou Lu por escrito que queria
agir rapidamente sobre esses defeitos. Em resposta, Lu passou e mexeu no vaso
sanitário e na pia, mas não conseguiu consertá-los. Ele disse que precisaria "ligar para
meu amigo Brian. Ele pode consertar essas coisas." Tom não podia cozinhar porque
não podia usar a pia e tinha que usar o banheiro do vizinho ou descer para o saguão se
o vizinho não estivesse em casa. Em 1º de dezembro, Lu ainda não havia resolvido os
problemas, então Tom não pagou seu aluguel. Janeiro chegou e os problemas ainda
não foram resolvidos, então Tom também não pagou o aluguel de janeiro. Lu
finalmente resolveu o problema em 15 de janeiro e agora quer que Tom pague o
aluguel atrasado. Supondo que o tribunal considere o contrato de locação válido, qual é
o valor mais provável que Tom precisará pagar a Lu pelo aluguel atrasado?

(A) O valor das instalações em seu estado defeituoso


(B) Tom não deve ter que pagar nenhum aluguel atrasado para Lu.

(C) O aluguel acordado menos o valor dos reparos necessários

(D) O valor total do aluguel menos dor e danos sofridos


Lógica:
A resposta correta é A. Embora os tribunais utilizem uma variedade de medidas de dano para uma
violação da garantia implícita de habitabilidade, a diferença entre o aluguel acordado e o valor do
imóvel em seu estado defeituoso é uma das mais comuns. Como as instalações ainda tinham
algum valor para Tom, ele provavelmente não será completamente dispensado de sua obrigação de
aluguel, então B está incorreto. C está incorreto porque nenhuma jurisdição usa essa fórmula;
certamente se Tom tivesse usado o remédio de reparo e dedução, seria apropriado, mas ele não
usou. As jurisdições também não costumam usar cálculos de dor e sofrimento, preferindo
considerar o valor diminuído das premissas como a medida adequada do dano a Tom. A resposta D
está, portanto, incorreta.
Leitura recomendada:
Páginas 341-342 de Dukeminier, Propriedade, Edição Concisa 2E
9

Paul possuía uma propriedade, Wedgewood, nas proximidades da Universidade. Seu


testamento dizia: "Venho por este meio conceber Wedgewood para minha filha Amélia
por 50 anos, depois para a Universidade." Qual das seguintes afirmações é
verdadeira?

(A) A universidade tem interesse executivo.

(B) A universidade tem um contingente remanescente.

(C) Amélia tem prazo de anos.

(D) O interesse da Universidade é nulo nos termos da Regra Contra as Perpetuidades.


Lógica:
A resposta correta é C. Amélia tem um patrimônio atual medido por um período de tempo
específico, de modo que é um prazo de anos. A é incorreta porque a Universidade tem um
interesse futuro em um terceiro após o término natural de um prazo de anos, e isso é chamado de
um remanescente, não um interesse executivo. A resposta B está incorreta porque o restante da
Universidade é investido e não contingente (sem condição precedente, sabemos quem é o
tomador). A resposta D também está incorreta porque a Regra Contra as Perpetuidades não se
aplica a nenhum dos juros aqui — lembre-se que ela se aplica apenas a remanescentes
contingentes e interesses executórios; A universidade tem um remanescente VESTED, então a
regra não se aplica.
Leitura recomendada:
Página 281 de Dukeminier, Propriedade, Edição Concisa 2E
10
Os irmãos Landon e Mica herdaram o pequeno negócio imobiliário de sua mãe em
Arcadia City. O portfólio dos irmãos inclui quatro casas unifamiliares e dois
apartamentos que muitas vezes são alugados como aluguel de temporada. Na tentativa
de conseguir alugar uma das casas, Landon fez uma postagem nas redes sociais,
afirmando:
"Casa unifamiliar para alugar a solteiros ou casais sem filhos."
Você é advogado da Procuradoria-Geral da República e percebe a postagem enquanto
percorre seu próprio feed. O post viola a lei federal de habitação justa?

(A) Sim, porque a Lei dos Direitos Civis de 1866 proíbe a publicidade discriminatória.

(B) Sim, porque a Lei de Habitação Justa proíbe postagens desse tipo.

(C) Não, porque a Lei dos Direitos Civis é inaplicável à habitação.

(D) Não, por causa de isenções legislativas às leis federais de habitação justa nos
EUA.
Lógica:
A resposta correta é (B). As postagens de mídia social são um anúncio, conforme definido pelo Fair
Housing Act, 42 U.S.C. 3601 et seq. A linguagem "para solteiros ou casais sem filhos" expressa
uma clara preferência por excluir famílias com filhos da propriedade, uma forma de discriminação
baseada no status familiar. (A) está incorreta porque, embora a CRA proíba a discriminação, suas
proteções não se estendem à publicidade. (C) está incorreta porque a Lei dos Direitos Civis se
aplica à moradia. (D) está incorreta porque os anúncios discriminatórios não estão sujeitos a
nenhuma das isenções da FHA (assim, os fatos relativos à carteira dos irmãos são irrelevantes e
não desencadeiam uma isenção).
Leitura recomendada:
Páginas 288-293 de Dukeminier, Propriedade, Edição Concisa 2E
11

Tabitha assinou um contrato de locação para alugar um apartamento de Linda semana


a semana, com o aluguel começando na segunda-feira, 2 de janeiro. Depois de 9
semanas, ela decidiu voltar a morar com a mãe. Ela pagou o aluguel na segunda-feira,
27 de fevereiro, para a semana seguinte. Ela disse a Linda na sexta-feira, 3 de março,
que iria se mudar no fim de semana e desistir do contrato. Ela entregou a Linda um
bilhete escrito nesse sentido. Linda respondeu que precisaria pagar a próxima semana
até domingo, 12 de março, também. Na ausência de qualquer autoridade estatutária
sobre o assunto, qual das seguintes afirmações é verdadeira?

(A) Tabitha deve aluguel a Linda por toda a próxima semana, até domingo, 12 de
março.

(B) Tabitha deve aluguel a Linda por parte da próxima semana, até sexta-feira, 10 de
março.
(C) Tabitha não deve a Linda nenhum aluguel adicional; ela está paga até esta semana
e deu a Linda o aviso prévio.

(D) Tabitha não deve a Linda nenhum aluguel adicional, porque o contrato de locação
não especificou o aviso prévio exigido.
Lógica:
A resposta correta é A. Em um contrato de locação periódica, o que claramente é, a common law
exige aviso prévio igual à duração do período — no caso, uma semana. C está, portanto, incorreto.
Além disso, a common law exige que a rescisão ocorra ao final do período — no caso, no domingo.
Portanto, neste momento, o aviso prévio de Linda só pode ser efetivado no final da próxima
semana, então ela deve aluguel por toda essa semana. Isso torna a resposta B incorreta. A
resposta D está incorreta; o contrato de locação não precisa especificar cláusulas de rescisão e
aviso prévio.
Leitura recomendada:
Páginas 281-282 de Dukeminier, Propriedade, Edição Concisa 2E
12

A fechadura da porta da frente da loja de roupas infantis de Hannah não funciona


corretamente desde que ela alugou o local há treze (13) anos. Ela informou o
proprietário sobre o bloqueio quebrado durante o primeiro ano de seu contrato de
locação, mas ele lhe disse: "Não é nada demais. Sinta-se livre para consertá-lo você
mesmo."
Hannah está frustrada com a atitude casual do proprietário e decidiu desocupar o local
este mês. Se ela desocupar este mês, vai ficar devendo mais algum aluguel?

(A) Sim, porque o senhorio não violou nenhum dos seus deveres ou obrigações.

(B) Sim, porque o senhorio violou o pacto do gozo tranquilo.

(C) Não, pois em uma ação de aluguel não pago, ela pode invocar a defesa de despejo
construtivo em razão da fechadura quebrada.

(D) Não, porque o senhorio violou a garantia implícita de habitabilidade.


Lógica:
A resposta correta é (A). Embora um locador residencial seja geralmente responsável por garantir
medidas básicas de segurança, como fechaduras de portas, o defeito aqui é pequeno e envolve
uma locação comercial. A boutique permaneceu aberta por treze anos, mesmo com a fechadura
quebrada, sugerindo que Hannah identificou um método alternativo de proteger as instalações. (B)
está incorreta porque sua lógica é internamente inconsistente. Se o proprietário violou o pacto de
gozo tranquilo, então Ana pode ser dispensada de sua obrigação de pagar aluguel. (C) é incorreto,
pois o despejo construtivo só ocorre quando ao locatário foi negado o usufruto benéfico do imóvel.
Aqui, Hannah permaneceu na posse e continuou operando sua loja por 13 anos com a fechadura
quebrada. Não lhe foi negado o gozo do imóvel. (D) está incorreta por duas razões alternativas:
primeiro, porque normalmente a garantia implícita não se aplica a locações comerciais. Em segundo
lugar, mesmo que essa jurisdição reconheça a garantia implícita de habitabilidade em locações
comerciais, a posse do espaço por 13 anos sugere que as instalações são adequadas para os fins
para os quais foi locado.
Leitura recomendada:
Páginas 322-344 de Dukeminier, Propriedade, Edição Concisa 2E
13

Torry foi enviado para o Afeganistão e decidiu desistir de seu apartamento, mas
precisava de um lugar para armazenar seu valioso Corvette enquanto ele estava fora.
Lisa escreveu a Torry oferecendo-se para alugar sua garagem a um aluguel de US $
300 por mês "durante a duração de sua implantação. Assinado, Lisa Landlord." Torry
ligou para Lisa imediatamente e aceitou a oferta. Após o primeiro ano, a unidade de
Torry foi considerada crucial para uma operação em andamento e sua implantação foi
estendida por mais doze meses. Lisa não está feliz porque agora planeja vender a casa
e quer encerrar o contrato de locação de Torry e tirar o carro da garagem. Torry se
recusa, dizendo que não pode lidar com isso de uma zona de guerra. Se Lisa tentar
rescindir o contrato, qual dos seguintes será o maior problema de Torry em mantê-la
presa?

(A) Um tribunal pode interpretar o arrendamento como um arrendamento à


vontade.

(B) Um tribunal pode interpretar o arrendamento como um prazo de anos.

(C) Um contrato de locação é automaticamente rescindido após a venda da taxa


subjacente simples.

(D) O Estatuto das Fraudes tornará a locação inexequível.


Lógica:
A resposta correta é A. Há alguma dúvida sobre qual é a categoria de locação é essa. Poderia ser
interpretado talvez como um período de anos, que dura por um determinado período de tempo –
neste caso, até que a guerra termine. Isso favoreceria Torry porque Lisa não poderia rescindir o
contrato antes do final desse período e tornaria B incorreto. No entanto, como a guerra não tem
uma data de término determinada, um tribunal pode interpretá-la como um arrendamento à vontade,
que normalmente é rescindível por qualquer uma das partes. Se assim for, Lisa poderia rescindir
(normalmente com um mês de antecedência). C está incorrecto porque o adquirente de um imóvel
está vinculado por quaisquer contratos de arrendamento prévios de que tenha conhecimento. D
está incorrecto porque a carta de Lisa é um memorando suficiente do acordo ao abrigo do Estatuto
das Fraudes, assinado pela parte a vincular. O fato de Torry não ter assinado não é relevante, a
menos que Lisa esteja tentando manter Torry em seus termos.

Capítulo 7

A concordou em vender Blackacre para B por US $ 200.000. Celebraram um contrato


de compra e venda exequível em 1 de janeiro de 2106, com data de encerramento em
1 de março de 2016, sujeito às habituais contingências de financiamento e título
negociável. No entanto, em 15 de fevereiro, A morreu. O testamento de A legou todos
os seus bens pessoais à sua filha, D, e atribuiu todos os seus bens imóveis ao seu
filho, S. Qual das seguintes afirmações é verdadeira?

(A) Se a jurisdição aplicar a conversão equitativa, S teria o direito mais forte ao produto
da venda.

(B) Se a jurisdição aplicar a conversão equitativa, D teria o direito mais forte ao


produto da venda.

(C) Independentemente de a jurisdição aplicar ou não a conversão equitativa, esta


venda pode ser anulada pelo executor, porque A morreu antes de fechar e o contrato
estava sujeito a contingências.

(D) A venda deveria ser realizada, mas na maioria das jurisdições, S e D teriam que
dividir o produto da ação, por agitação.
Lógica:
A resposta correta é B. Essa questão exige que você considere a aplicação da conversão equitativa
e, claro, entenda a diferença entre propriedade pessoal e propriedade real. Se o tribunal aplicasse a
conversão equitativa, trataria a venda como concluída quando o contrato fosse executado, de modo
que B era o proprietário equitativo da Blackacre em 1º de janeiro. Nesse ponto, A teria apenas um
direito equitativo ao produto da venda, que é propriedade pessoal. Portanto, se a jurisdição aplicar a
conversão equitativa, D, o legatário, teria a pretensão mais forte. A resposta A é, portanto, incorreta
porque deturpa o resultado da aplicação da conversão equitativa. A resposta C está incorreta
porque os contratos imobiliários válidos e exequíveis não são afetados pela morte de uma das
partes. Além disso, mesmo que haja contingências, desde que sejam as contingências normais, os
tribunais aplicarão a conversão equitativa. A resposta D está incorreta porque o testamento
especifica claramente qual beneficiário tem direito a qual tipo de imóvel; Não há fundamento para
que o tribunal se afaste da intenção do testador.
Leitura recomendada:
Página 370 de Dukeminier, Propriedade, Edição Concisa 2E
2

Alf Landon deu ao First Bank uma hipoteca sobre Blackacre em troca de um
empréstimo de US$ 50.000 em uma jurisdição de notificação. O First Bank registrou a
hipoteca, mas o escritório do registrador indexou erroneamente a hipoteca sob Alf
"London" em vez de "Landon". Um ano depois, Alf vendeu o imóvel para Betty por
escritura de garantia geral. Ela imediatamente o revendeu para Carl usando uma
escritura de garantia especial. Carl transferiu-o vários dias depois para Dana por
escritura de desistência. Alguns meses depois, Alf deu calote no empréstimo e o First
Bank iniciou um processo de execução contra a Blackacre. Qual das seguintes opções
é provavelmente verdadeira na maioria dos estados?

(A) O First Bank não pode executar sua hipoteca contra a Dana, porque a hipoteca não
foi devidamente registrada.
(B) A hipoteca ainda é boa, e Dana deve ser capaz de processar Betty com base em
garantias na escritura que Betty deu a Carl.

(C) A hipoteca ainda é boa, e Dana deve ser capaz de processar Alf com base no pacto
contra gravames.

(D) A hipoteca ainda é boa, e Dana deve poder processar Alf sob o pacto de gozo
tranquilo.

Lógica:
A resposta correta é D. Essa pergunta exige que você considere se as garantias de título "correm
com a terra" para os proprietários subsequentes. Primeiro, no entanto, você tem que considerar se
a hipoteca do First Bank ainda é válida. Foi registrado em tempo hábil, mas indexado
incorretamente. A maioria das jurisdições sustenta que a indexação incorreta, se não for culpa da
pessoa que grava, ainda se qualifica como notificação de registro, embora torne a escritura difícil de
descobrir em uma pesquisa normal de título. Nesse caso, isso significa que o First Bank ganha
porque fez o que deveria fazer: registrar a hipoteca. Assim, a resposta A está incorreta. Dana,
portanto, precisa recuperar seus danos contra alguém. Ela não pode processar Carl com base nas
garantias porque ele deu apenas uma escritura de desistência, que não tem garantias. Betty deu
apenas uma escritura de garantia especial, que garante apenas contra defeitos que surgiram
enquanto Betty possuía Blackacre. Esse defeito ocorreu antes de ela obter o título, então Dana não
pode processar Betty. A resposta B está, portanto, incorreta. Um processo contra Alf é possível
porque Alf deu a Betty uma escritura de garantia geral. A regra geral é que as garantias atuais de
título não correm para os proprietários subsequentes, enquanto as garantias futuras o fazem.
Portanto, como o pacto contra ônus é garantia presente, a resposta C está incorreta. O pacto de
gozo tranquilo, responde D, é uma garantia futura e corre para compradores subsequentes como
Dana. Ela só é violada quando a posse de Dana é perturbada por alguém que reivindica um título
superior, o que descreve com precisão a execução hipotecária pelo First Bank neste caso. Portanto,
a resposta D é mais provável de ser o resultado na maioria dos estados.
Leitura recomendada:
Páginas 388-394 de Dukeminier, Propriedade, Edição Concisa 2E
3

Alf quer que sua filha Becky tenha Blackacre quando ele morrer. Ele redigiu uma
escritura para Blackacre que diz que "Alf concede Blackacre a Becky, seus herdeiros, e
atribui para sempre". No aniversário dela, ele mostra a escritura e diz a ela que estará
em uma gaveta em sua mesa com seu testamento, que dá todos os seus bens para os
Odd Fellows, uma organização de caridade. Quando Alf morre, seu executor encontra
a escritura e a entrega a Becky, que prontamente a registra. Becky é dona do
Blackacre?

(A) Não, porque não houve contraprestação pela escritura

(B) Não, porque o testamento tem precedência sobre qualquer transmissão do bem por
doação, salvo se a escritura de doação for lavrada antes do falecimento do outorgante
(C) Sim, porque a doação ocorreu antes da morte de Alf, embora só tenha sido
registrada mais tarde

(D) Não, porque a escritura não foi efetivamente entregue

Lógica:
A resposta correta é D. Essa pergunta exige que você considere a exigência de entrega de
escrituras. Alf pode ter tido a intenção adequada de dar Blackacre a Becky, mas uma escritura deve
ser entregue para ser eficaz. A entrega exige que Alf abra mão do controle sobre a escritura, o que
ele nunca fez. Ele poderia ter mudado de ideia, tirado a escritura da gaveta e destruído. O caso
Rosengrant v. Rosengrant discute a entrega de escrituras e a necessidade de abrir mão do
controle. A resposta A está incorreta porque um ato não requer consideração para ser eficaz. A
resposta B também está incorreta porque uma escritura não precisa ser registrada para ser
efetivada. É eficaz na entrega, embora o registro certamente possa ajudar a estabelecer que a
escritura foi entregue e quando. Como discutido acima, a resposta C está incorreta porque a
doação não foi concluída antes da morte de Alf. Como a escritura não foi efetivamente entregue
antes da morte de Alf, os Odd Fellows tomarão Blackacre de acordo com o testamento.
Leitura recomendada:
Páginas 394-397 de Dukeminier, Propriedade, Edição Concisa 2E
4

Amy vive no lado sul da Rota 1, do outro lado da estrada de Jan e Mike Smith, um
casal, que vivem no lado norte. Um dia ela estava tomando café com os Smiths em seu
convés traseiro, e Mike mencionou que estava pensando em vender Blackacre, cerca
de 20 acres de terra adjacentes à terra de Amy no lado sul da Rota 1, que ele herdou
de seu tio. Amy sempre quis expandir sua operação de laticínios e disse aos Smiths
que queria comprar a terra. Amy afirma que eles concordaram com um preço de US $
4000 / acre, com o preço exato a depender de um levantamento da terra para
determinar a área da parcela. Amy anotou em um pedaço de papel: "Amy Johnson,
compradora, e Jan e Mike Smith, vendedores/proprietários. Propriedade: 20 hectares,
mais ou menos, no lado sul da Rota 1, Condado de Pawnee. Preço: $4000/acre.
Pesquisa a ser paga por Amy." Amy e Mike assinaram este artigo. Jan não assinou.
Amy afirma que Jan disse: "Esse é o departamento de Mike. Nunca tive nada a ver com
aquela terra. Tudo o que ele quiser está bem." Depois disso, Amy contratou um
topógrafo, e Jan se reuniu com o topógrafo para mostrar-lhe onde estava localizada a
propriedade a ser pesquisada. A pesquisa custou US $ 1000 e mostrou que o terreno
tinha cerca de 21 hectares. Amy contratou um advogado para buscar o título para
Blackacre, que lhe custou US$ 500. A pesquisa de título mostra que o terreno é detido
por Mike e Jan Smith, como inquilinos por inteiro. Ela também pagou US$ 800 por
alguns reparos na cerca para que a terra estivesse pronta para pastar seu gado
imediatamente após o fechamento. No fechamento, Mike disse que o negócio teria que
ser cancelado porque Jan se recusou a assinar a escritura, reclamando que nunca quis
vender e acha que o terreno valerá mais em um futuro próximo. Qual das seguintes
afirmações é verdadeira?

(A) Amy pode processar Jan por desempenho específico, porque o pedaço de papel
atende ao Estatuto de Fraudes.
(B) Amy não pode processar Jan por desempenho específico, porque o pedaço de
papel não contém uma descrição legal correta de Blackacre.

(C) Amy não pode processar Jan por desempenho específico, porque o pedaço de
papel não contém o preço exato do imóvel.

(D) Amy deve ser capaz de manter Jan no contrato, apesar do Estatuto de
Fraudes, sob uma teoria de preclusão ou execução parcial.

Lógica:
A resposta correta é D. Para contratos imobiliários, o Estatuto das Fraudes exige que haja um
memorando do acordo assinado pela parte a ser vinculada, mostrando o preço, o imóvel e as
partes. O pedaço de papel, neste caso, tem uma descrição adequada do imóvel; não precisa ser
uma descrição legal, mas apenas suficiente para que um tribunal determine qual propriedade foi
destinada. Neste caso, o imóvel estava suficientemente descrito. Portanto, a resposta B está
incorreta. Além disso, embora o papel não indique o preço exato, ele é suficiente desde que haja
um método pelo qual o preço possa ser determinado. Portanto, a resposta C está incorreta porque o
método de cálculo do preço está suficientemente definido. No entanto, o Estatuto das Fraudes não
é cumprido aqui porque Jan não o assinou e ela é a "parte a ser vinculada". Como o imóvel é
mantido como locação por inteiro, o negócio não pode passar sem ela. Portanto, a resposta A
também está incorreta. No entanto, Amy gastou dinheiro em confiança razoável no acordo de Jan
para o acordo. Seus gastos com o levantamento, o trabalho de titulação e os reparos da cerca
devem ser suficientes para estabelecer o desempenho do estoppel ou da peça. Jan sabia que Amy
estava gastando o dinheiro, mas ficou permitindo que isso acontecesse.
Leitura recomendada:
Páginas 362-366 de Dukeminier, Propriedade, Edição Concisa 2E
5

Betty Buyer comprou uma casa de Sally Seller, tomando o título por escritura de
garantia geral. A jurisdição não tem requisitos legais de divulgação sobre o estado das
instalações. Comprador inspecionou o imóvel antes de fechar, mas não encontrou nada
de errado. Pouco depois de se mudar, no entanto, ela começou a se sentir mal. Depois
de alguns meses, ela descobriu que havia crescimento de mofo por trás dos painéis na
sala de estar. A vendedora havia colocado o painel pouco antes de vender o imóvel e
admite que viu algum mofo na época. O comprador recebeu uma estimativa de US$
20.000 para corrigir o problema. O comprador agora quer recuperar esse valor em
danos do vendedor. Qual das seguintes afirmações é verdadeira?

(A) Como a regra geral é caveat emptor, o Comprador não pode recuperar danos.

(B) O comprador pode recuperar danos, porque o defeito não foi detectável por
inspeção razoável.

(C) O comprador pode recuperar danos, porque a propriedade deve estar livre de todos
os defeitos materiais.
(D) O comprador pode recuperar danos com base na declaração falsa afirmativa do
vendedor.
Lógica:
A resposta correta é B. Um vendedor deve divulgar defeitos materiais que são conhecidos por eles
e não são detectáveis após uma inspeção razoável. Trata-se de uma exceção à regra da caveat
emptor, tornando a resposta A incorreta. O molde era um defeito material, pois teria impacto no
valor do imóvel. Não foi descoberto porque uma inspeção razoável não incluiria rasgar painéis. Era
conhecido da vendedora porque ela admite isso. Portanto, a resposta B está correta. A resposta C
é muito ampla; certamente a propriedade não precisa estar isenta de defeitos materiais. Pode haver
defeitos, desde que sejam patenteados ou divulgados pelo vendedor, para que os compradores
possam considerá-los no preço da oferta. A resposta D está errada porque não temos provas de
que o vendedor tenha feito qualquer declaração em relação ao mofo ou à qualidade geral da casa.
Leitura recomendada:
Páginas 371-378 de Dukeminier, Propriedade, Edição Concisa 2E
6

O comprador celebrou um contrato para comprar um rancho de Seller por US $ 1


milhão. O rancho consistia de uma antiga casa e celeiro, bem como um estábulo para
cavalos e algumas terras de pastagem. O contrato especificava na Cláusula 1 que era
"dependente da obtenção de financiamento aceitável pelo Comprador". O contrato não
continha cláusula de vistoria, mas especificava que o comprador estava comprando o
imóvel "como está". O negócio estava previsto para ser fechado em 60 dias. Durante o
período de fechamento, o comprador descobriu que o celeiro tinha tapumes
apodrecidos e provavelmente precisaria ser completamente derrubado em um futuro
próximo. A jurisdição não tem divulgação obrigatória para esse defeito. O Comprador
solicitou que o Vendedor corrigisse esse defeito ou reduzisse o preço em US$ 100.000.
Vendedor recusou. A compradora notificou então a vendedora de que estava
rescindindo o contrato com base no defeito do celeiro e em sua falha em obter
financiamento. Compradora indicou que fez vários pedidos a vários bancos. Dada a
sua classificação de crédito, apenas uma concordou em financiar o negócio, mas a
apenas 80% do preço de compra, a uma taxa acima da taxa normal para uma hipoteca
de 30 anos. O vendedor busca a execução específica do negócio. Qual dos itens a
seguir é provavelmente verdadeiro?

(A) O comprador deve poder rescindir com base no defeito do celeiro, mas não no
financiamento.

(B) O comprador deve poder rescindir com base no financiamento, mas não no
defeito do celeiro.

(C) O comprador deve poder rescindir com base no financiamento ou no defeito do


celeiro.

(D) O comprador não poderá rescindir com base no financiamento ou no defeito do


celeiro.
Lógica:
A resposta correta é B. No que diz respeito ao defeito do celeiro, o dever do vendedor limita-se a
revelar quaisquer defeitos materiais de que tenha conhecimento que não seriam revelados após
uma inspeção razoável. O defeito do celeiro parece ser facilmente detectável, por isso não é um
defeito latente que precisava ser divulgado. O contrato imobiliário não continha cláusula de
fiscalização; em vez disso, o comprador concordou em aceitar as instalações "como estão". Isso
significa que ela concordou em aceitar quaisquer defeitos patenteados, como o tapume apodrecido
do celeiro. Isso torna as respostas A e C incorretas. No que diz respeito ao financiamento, o
contingenciamento é escrito de forma muito ampla, pois não especifica os parâmetros de
financiamento (taxa de juros, entrada, etc.) que o Comprador deve aceitar. Portanto, o dever do
comprador se limita a fazer "melhores esforços" para encontrar financiamento aceitável, o que ela
aparentemente fez. Essa contingência amplamente escrita deve permitir que ela desista do negócio
e, portanto, a resposta D não pode estar correta.
Leitura recomendada:
Páginas 371-378 de Dukeminier, Propriedade, Edição Concisa 2E
7

Fausto queria comprar o lote 37 da Berkley Hills Addition de Beowulf. A propriedade


contém uma casa de dois andares com uma garagem independente. O endereço da
propriedade é 1839 Larson Street. Fausto fez um pequeno tour pela casa e adorou
imediatamente. Fausto estava se preparando para partir para a Europa e os dois não
tinham um contrato de formulário prontamente disponível, então Beowulf simplesmente
escreveu os termos em um pedaço de papel da seguinte forma: "Fausto concorda em
comprar 1839 Larson Street de Beowulf por US $ 100.000. Beowulf concorda em
fornecer título comercializável. Fechando para daqui a dois meses." Ambas as partes
assinaram e dataram o documento. Ao retornar da Europa várias semanas depois,
Fausto deu uma olhada mais de perto na casa e descobriu gesso do teto manchado e
dobrado em um quarto, indicando que o telhado vaza mal. Agora, ela quer desistir do
contrato. Na ausência de quaisquer requisitos legais, qual dos seguintes requisitos é
verdadeiro?

(A) O contrato inicial de compra e venda foi insuficiente para vincular Fausto a essa
compra.

(B) O telhado vazado constitui violação de título comercial, pois tem impacto material
no valor do imóvel, permitindo a Fausto rescindir.

(C) O telhado vazado deveria ter sido revelado a Fausto, para que ela possa agora
rescindir.

(D) Fausto não tem fundamento válido para a rescisão.

Lógica:
A resposta correta é D. A resposta A está incorreta porque o memorando escrito do contrato atende
aos requisitos do Estatuto de Fraudes: Preço, Partes, Propriedade. O preço e as partes estão
claramente indicados; A única questão é a descrição do imóvel. Embora o endereço da rua não seja
uma descrição legal correta, é suficiente para o contrato, pois é claro qual o imóvel pretendido. A
resposta B está incorreta porque um defeito físico no imóvel nada tem a ver com título comercial. A
resposta C tem a ver com a doutrina da common law de caveat emptor. Na common law, os
compradores assumiram o imóvel como está e assumiram o risco de quaisquer defeitos. As
modificações modernas exigem a divulgação de defeitos latentes, que são problemas que causam
impacto material no valor do imóvel. O telhado vazado é um defeito material, mas não é latente. Um
defeito latente é aquele que é conhecido pelo vendedor, mas não pode ser descoberto por uma
inspeção razoável. Esse defeito era patente; era fácil para Fausto ver que o gesso estava
manchado e flambando. Portanto, Fausto não pode rescindir com base nisso. Normalmente, os
contratos imobiliários contêm uma cláusula de inspeção que permite ao comprador um certo
período de tempo para uma inspeção mais detalhada para encontrar defeitos físicos. Esse contrato,
no entanto, não incluía tal cláusula. Além disso, muitos estados têm estatutos que exigem a
divulgação de certos defeitos materiais; O problema pede que você ignore essa possibilidade. Por
isso, Fausto está preso à sua pechincha.
Leitura recomendada:
Páginas 362-379 de Dukeminier, Propriedade, Edição Concisa 2E
8

Em 2000, Andrew transmitiu Blackacre para sua filha Beatrice, sem nenhuma
consideração, por escritura de desistência. Em 2005, Beatrice transmitiu Blackacre
para Conner por US $ 500.000 por escritura de garantia geral. Em 2015, Conner queria
vender Blackacre para Drake. Durante o processo de busca pelo título, Drake
descobriu que Andrew era casado com Winona na época da transferência de Andrew
para Beatrice, embora Blackacre fosse intitulado apenas em nome de Andrew. Winona
não participou da transmissão de forma alguma e ainda está viva. Esta jurisdição prevê
um interesse estatutário de um terço em qualquer propriedade do outro cônjuge
durante o casamento. Embora Winona não tenha declarado seu interesse e Andrew
ainda esteja vivo, Drake se recusa a pagar mais de US $ 400.000 pela propriedade.
Conner pode provar que o valor justo de mercado da propriedade é de US $ 600.000 e
esse é o valor que Drake concordou em pagar antes que a pesquisa do título revelasse
o potencial interesse de Winona. A jurisdição tem um prazo de prescrição de cinco
anos para reclamações baseadas em violação de garantias de título. Qual das
seguintes afirmações é verdadeira na maioria das jurisdições?

(A) Conner pode processar Andrew, porque ele não conseguiu obter uma liberação do
interesse de Winona na transmissão para sua filha Beatrice.

(B) Conner pode processar Beatriz sob o pacto de seisin.

(C) Conner pode processar Beatriz sob o pacto de garantia geral.

(D) Conner não tem nenhuma reivindicação viável contra qualquer parte no
momento.

Lógica:
A resposta correta é D. Essa pergunta exige que você considere o efeito das garantias nas
escrituras e a natureza do dower. A resposta A está claramente errada. Mesmo que essa questão
do título seja "culpa" de A, de certa forma, ele não deu nenhuma garantia de título. Uma escritura de
desistência basicamente diz: "qualquer interesse que eu possa ter é seu". Não faz promessas
respeitando a qualidade do título. Beatriz, no entanto, deu a Conner uma escritura de garantia geral.
Isso contém convênios presentes e futuros. O atual pacto de seisin foi violado no momento da
transmissão porque Beatriz não tinha bom título. No entanto, a violação ocorreu em 2005 e o prazo
prescricional já correu sobre essa alegação. Portanto, a resposta B está incorreta. Garantias
futuras, como o pacto de garantia geral, são violadas apenas quando o garantidor (Conner) é
perturbado em sua posse por alguém que reivindica título superior. Os fatos indicam que Winona
não reivindicou seu interesse, nem poderia, porque é incólume até que Andrew morra. A resposta C
não pode estar correta, portanto. Assim, a resposta correta é D porque atualmente Conner não
pode processar ninguém por esse defeito de título.
Leitura recomendada:
Páginas 388-394 de Dukeminier, Propriedade, Edição Concisa 2E
9

Joylon quer dar Bleak House para seu filho Junior. Ele escreve em um pedaço de
papel: "Escritura: Eu, Joylon, expresso minha intenção de dar a meu filho Junior a
propriedade conhecida como Bleak House, legalmente descrita como Lote 1 da
Subdivisão de Copperfield, Condado de Dickens, Kansas." Ele assina esse papel e
entrega para o Junior. Vários dias depois, Joylon morre, deixando todos os seus bens
para sua filha Estelle por testamento. A jurisdição não permite testamentos
holográficos, por isso Junior alega que o papel que lhe foi dado foi uma escritura. Em
uma ação movida por Estelle para reivindicar Bleak House, qual é sua melhor alegação
sobre a validade da tentativa de transmissão para Junior?

(A) O papel não foi autenticado em cartório.

(B) O papel não foi assinado por Junior.

(C) O jornal não utilizou linguagem adequada de veiculação.

(D) Joylon deveria ter usado um formulário de escritura oficial.


Lógica:
A resposta correta é C. O artigo não utilizou linguagem de veiculação atual. Joylon manifestou a
intenção de transmitir a propriedade para Junior no futuro. Uma escritura apropriada diria que
Joylon "concede, transfere e transmite" a propriedade, ou mais simplesmente, apenas "transmite",
enquanto essa linguagem indica uma intenção de fazê-lo, mas na verdade não o faz. É possível que
Junior consiga convencer um tribunal a ignorar esse defeito, mas é provavelmente o melhor
argumento de Estelle. Sem uma transmissão adequada durante a vida, Joylon ainda é o dono da
Bleak House e o título passa sob sua vontade para Estelle. Além das palavras de transmissão, os
elementos de uma escritura incluem: a descrição do imóvel, a identificação do concedente e do
donatário e a assinatura do concedente. A validade de uma escritura não depende de
reconhecimento de firma, embora normalmente os estatutos exijam o reconhecimento de firma para
o registro da escritura. Portanto, a resposta A está incorreta. A resposta B está errada porque o
donatário não assina uma escritura. É uma ação do concedente; portanto, a assinatura do donatário
seria supérflua. Por fim, a resposta D está incorreta, pois não é necessário que o concedente utilize
algum tipo de formulário oficial; uma escritura pode ser escrita em um pedaço de papel (embora não
recomendado!).
Leitura recomendada:
Páginas 388-390 de Dukeminier, Propriedade, Edição Concisa 2E
10
O queria dar Blackacre para sua filha, A, quando ele morresse. Ele escreveu uma
escritura transmitindo Blackacre a A e a entregou a A afirmando: "Eu quero que você
tenha isso no caso de eu morrer". A guardar a escritura no armário. Mais tarde, A
dirigiu-se a O e disse que gostaria que O acrescentasse o seu filho B à escritura. O
concordou com essa mudança. Retomou a escritura e escreveu em nome de B para
que agora se lesse "a A e B como condôminos". A novamente colocou a escritura no
armário. Ó então morreu, deixando um testamento entregando todos os seus bens à
Igreja. A pegou a escritura e mostrou ao testamenteiro de O. Qual é o estado mais
provável do título de Blackacre, no common law?

(A) A Igreja é proprietária da Blackacre, porque a escritura nunca foi devidamente


entregue.

(B) A é dono da Blackacre, pois a adição posterior do nome de B foi ineficaz.

(C) A e B possuem Blackacre como condôminos.

(D) A Igreja é proprietária da Blackacre, porque a escritura de A não foi registada e o


posterior aditamento de B invalidou a escritura.
Lógica:
A resposta correta é B. O executou uma escritura a A e entregou-a a ela. A entrega da escritura
está em causa porque O parecia anexar uma condição oral à entrega. Na common law, tais
condições orais não são válidas porque há muita chance de fraude, uma vez que a condição é
incompatível com o rosto do ato. Alguns tribunais invalidaram a entrega em vez da condição, mas
na common law, a entrega seria mantida. Assim, a resposta A está incorreta. O abriu mão do
controle sobre a escritura, ao contrário da escritura testamentária em Rosengrant. Portanto, O
corretamente transmitiu Blackacre a A. Quando A devolveu a escritura a O para que pudesse
acrescentar B, O já não possuía Blackacre; portanto, sua alteração da escritura não surtiu efeito.
Embora não tenha surtido efeito, não invalidou o ato, como sugere a resposta D. Uma vez que a
escritura faz seu trabalho de transmitir título de O para A, realmente não importa, legalmente, o que
acontece com a escritura – a escritura, com efeito, está feita. É claro que, do ponto de vista prático,
queremos preservar a escritura e registrá-la imediatamente para avisar; no entanto, juridicamente
falando, fez o seu trabalho de transmitir o título de O para A. Para acrescentar B como condómino,
A deveria ter transmitido, com nova escritura, "a A e B como conarrendatários" (ou, em algumas
jurisdições, transmitido a um arrendatário primeiro para conhecer as unidades de tempo e título).
Como isso não foi feito, A continua sendo o único proprietário de Blackacre na morte de O, e a
resposta C está incorreta.
Leitura recomendada:
Páginas 362-366 de Dukeminier, Propriedade, Edição Concisa 2E
11

Sayers firma um contrato para vender Blackacre, uma parcela de 80 acres em uma
área rural, para Byers. O contrato diz que Sayers fornecerá "título comercializável,
sujeito a todas as servidões e ônus de registro para Blackacre, legalmente descrito
como W1/2 SE1/4 Seção 16 no Condado de Guthrie". Ao examinar o título, Byers
descobre que Blackacre está onerado por uma servidão de gasoduto. Além disso,
verifica-se que Sayers não tem registro de propriedade de cerca de cinco hectares de
terra (Parcela A) na borda do imóvel descrito no contrato. Sayers mostra a Byers
evidências, no entanto, de que ele está na posse contínua e exclusiva da Parcela A há
mais de 20 anos. Dado que o período de usucapião no estado é de apenas sete anos,
Sayers diz que estabeleceu um bom título para a Parcela A. Byers se recusa a fechar,
alegando uma falha de título comercializável. Qual das seguintes afirmações é
verdadeira?
C
(A) Tanto a servidão quanto a emissão da Parcela A violam as disposições do título
comercializável deste contrato.

(B) A servidão viola a previsão de título negociável do contrato; a questão da Parcela A


não.

(C) A servidão não viola a previsão de título negociável do contrato; a questão da


Parcela A sim.

(D) Nem a servidão nem a emissão da Parcela A violam a previsão de título negociável
do contrato.
Lógica:
A resposta correta é C. O título mercantil exige que a vendedora comprove que é a proprietária do
registro, em taxa simples absoluta, do imóvel descrito, sem ônus. Nesse caso, há duas possíveis
violações dessa norma: a servidão e a não efetivação do registro de titularidade da Parcela A. De
acordo com o common law, ambas as questões violariam o padrão de título comercializável. No
entanto, neste caso, Byers optou por aceitar um contrato com termos ligeiramente diferentes, uma
promessa de fornecer "título comercializável sujeito a todas as servidões e ônus de registro". Como
a servidão do gasoduto está registrada, ela não viola as disposições de título comercializável deste
contrato. Assim, as respostas A e B estão incorretas. A questão da Parcela A, no entanto, não é
abrangida por esta exceção contratual. Mesmo que Sayers possa, de fato, ter o título da Parcela A,
ela não tem "título comercializável" porque o título comercializável não pode estar sujeito a uma
perspectiva não frívola de litígio sobre a propriedade. Isso torna a resposta D incorreta. Sayers
precisa silenciar o título da Parcela A antes de tentar vendê-la ou obter um comprador em potencial
para torná-la uma exceção específica à disposição de título comercializável.
Leitura recomendada:
Páginas 366-370 de Dukeminier, Propriedade, Edição Concisa 2E
12

Stephen Gates comprou a Blueacre em 2001 por US$ 10 milhões. Ele pagou US$ 1
milhão em dinheiro e financiou o restante com um empréstimo do First Bank (FB)
dando ao FB uma hipoteca que foi prontamente registrada. Em 2005, Gates contraiu
outro empréstimo, desta vez do Second Bank (SB), que o garantiu com uma hipoteca
sobre a Blueacre, também prontamente registrada. Pouco depois, Gates deixou de
pagar seus empréstimos. Ele ainda devia US$ 7 milhões ao FB e US$ 1 milhão ao SB.
Os dois bancos fecharam a Blueacre e o tribunal realizou a venda hipotecária. O único
licitante na venda foi Bill Jobs, que comprou a propriedade por US$ 6 milhões. Qual
das seguintes afirmações é verdadeira?
(A) Se este estado tiver promulgado legislação antideficiência, FB e SB devem ser
capazes de processar Gates pelo restante do que ele deve.

(B) Na maioria dos estados, a venda poderia ser anulada se o tribunal não tomasse
medidas suficientes para alcançar um preço justo para o imóvel.

(C) Mesmo após a venda da hipoteca para Jobs, alguns estados permitiriam que
Gates resgatasse a propriedade pagando o preço de venda da hipoteca de US$ 6
milhões.

(D) A SB deve ter direito a uma parte proporcional dos US$ 6 milhões de proventos.
Lógica:
A resposta correta é C. Esta pergunta testa o seu conhecimento de vários aspectos do processo de
execução hipotecária, especificamente o direito de resgate e a prioridade de pagamento entre as
hipotecas. Neste cenário, há uma primeira hipoteca e uma segunda hipoteca que tem uma
prioridade menor. A resposta A está incorreta porque a legislação antideficiência limita a hipoteca
ao imóvel garantidor da dívida; portanto, nessas jurisdições, os credores não poderiam processar
Gates pelos valores restantes devidos. Com relação à resposta B, em algumas jurisdições, as
hipotecas que realizam execuções extrajudiciais são obrigadas a tomar medidas para garantir um
preço justo. No entanto, trata-se de uma execução judicial, conduzida pelo próprio tribunal, pelo que
a única questão seria se o tribunal seguiu os procedimentos legais. Não há indicação no problema
de que os requisitos legais não foram seguidos. A resposta C refere-se ao direito legal de resgate
previsto em alguns estados, que permite ao devedor resgatar o imóvel dentro de um determinado
período de tempo após a venda hipotecária. A resposta C afirma corretamente a lei. A resposta D
está incorreta porque a prioridade dos penhores determina quem recebe primeiro. Nesta situação, o
First Bank tem prioridade e seria pago completamente antes que o Second Bank recebesse
qualquer coisa. Não há partilha proporcional dos proventos da execução hipotecária.

Capítulo 8

Em 2010, Whistler pegou US$ 50 mil emprestados de sua mãe, Ma, que morava nas
proximidades. Ele assinou uma nota e uma hipoteca sobre a Blackacre, um imóvel
alugado que possuía, para garantir o empréstimo. A Maçã, no entanto, não registrou a
hipoteca naquele momento. Em 1 de março de 2016, Whistler celebrou um contrato
para vender a Blackacre para Bono. Bono realizou uma pesquisa de título que, claro,
não revelou a hipoteca para Ma e ele não foi informado de seu interesse. Em 1º de abril
de 2016, a venda foi encerrada. Bono pagou o valor total do imóvel (US$ 200 mil) em
troca de uma escritura de garantia geral. Whistler disse a Ma que a pagaria com o
produto da venda, mas não o fez. Em vez disso, pegou o dinheiro e partiu para a
Europa. Em 15 de abril de 2016, Ma registrou sua hipoteca. Em 30 de abril de 2016,
Bono registrou a escritura geral de garantia. Ma agora tenta abrir mão de Blackacre
para satisfazer sua hipoteca. Qual das seguintes afirmações é verdadeira?
(A) Se este estado tiver um ato de registro de aviso de corrida, Ma não poderá
renunciar porque seu interesse não foi registrado quando Bono comprou a Blackacre.

(B) Se o estado tiver um ato de gravação do tipo aviso, Ma não poderá executar
Blackacre, mesmo que ela tenha gravado primeiro.

(C) A menos que o estado tenha um ato de registro de raça puro, Ma não poderá se
excluir de Blackacre.

(D) Independentemente do tipo de ato de gravação, o interesse da Maçã não


sobreviveria à transferência de titularidade para Bono; seu único remédio é processar
Whistler na nota.
Lógica:
A resposta correta é B. Em um problema de ato de gravação, desenhe um pequeno diagrama
mostrando quando as transmissões e gravações ocorreram. Neste caso, Whistler transmitiu para
Ma, depois Whistler transmitiu para Bono, depois Ma gravou, depois Bono gravou. Como a
transmissão para Ma foi a primeira no tempo, Ma poderá renunciar a menos que Bono satisfaça os
requisitos do ato de gravação da jurisdição. Se for um ato de notificação, Bono ganha porque tomou
sem aviso prévio da transação anterior. Se for um ato de aviso de corrida, Bono perde porque,
mesmo tendo tomado sem aviso prévio, ele também deve gravar primeiro, o que ele não fez. Da
mesma forma, sob um estatuto de raça pura, Bono perde porque Ma gravou primeiro. Portanto, a
resposta A está incorreta porque a Maçã seria capaz de fechar em um estado de aviso de corrida.
As respostas C e D também estão incorretas pelo mesmo motivo. A resposta B é a única afirmação
correta da lei.
Leitura recomendada:
Páginas 437-439, 446-451 de Dukeminier, Propriedade, Edição Concisa 2E
2

Joy era dono de uma casa chamada Greenacre. Quando sua mãe, Dawn, ficou frágil,
Joy a convidou para morar com ela em Greenacre. Ela redigiu uma escritura
concedendo à mãe um interesse de vida em Greenacre e a deu a ela, afirmando "se
algo acontecer comigo, quero que você possa viver aqui para sempre". Dawn pegou a
escritura e a colocou na gaveta de sua mesa. Pouco tempo depois, Joy se apaixonou
por Rock e se casou com ele. Ela citou Greenacre "de Joy a Joy e Rock como
inquilinos conjuntos com direito de sobrevivência". Ela disse a Rock: "Claro, isso está
sujeito ao direito de minha mãe de viver sua vida aqui; Dei-lhe um interesse de vida."
Rock disse: "Claro, sem problemas", e imediatamente registrou essa ação.
Tragicamente, Joy foi morta alguns meses depois em uma avalanche enquanto
esquiava. Rock não tem intenção de viver em Greenacre com sua sogra. Ele decidiu
vender Greenacre para seu amigo Hardplace. Ele disse a Hardplace: "Olha, Dawn tem
um patrimônio vitalício, aparentemente, mas como não está gravado, você deve ser
capaz de se livrar dela imediatamente". Ele transmitiu à Hardplace por escritura de
quitação pelo valor justo de mercado total da Greenacre, e a Hardplace imediatamente
registrou essa escritura. Qual dos seguintes itens é verdadeiro em um processo de
Hardplace para expulsar Dawn de Greenacre?

(A) A Hardplace vencerá apenas em um ato de registro de raça pura.


(B) A Hardplace perderá apenas em um ato de registro de notificação jurisdicional.

(C) A Hardplace perderá em qualquer jurisdição.

(D) A Hardplace vencerá em qualquer jurisdição.


Lógica:
A resposta correta é A. Parece que Dawn está entre Rock e Hardplace, mas será que ela é
mesmo? A alegria transmitiu corretamente um interesse de vida a Dawn. Ela então transmitiu um
meio interesse com direito de sobrevivência para Rock, mas como a transmissão para Dawn veio
primeiro, no interesse de Rock é onerado pelo patrimônio vitalício. Quando Dawn morre, Rock tem a
taxa simples, mas ainda é onerado pelo patrimônio vitalício anteriormente transmitido. Quando ele
transmite a Hardplace, no common law, ele permanece onerado pelo patrimônio vitalício. Portanto,
a única maneira que Hardplace pode se livrar do interesse de Dawn é entrando nos termos do ato
de gravação. Em uma jurisdição de notificação ou aviso de corrida, um comprador subsequente
pelo valor tem que tomar sem aviso prévio; Hardplace tinha notado aqui porque foi informado do
interesse de Dawn. Assim, a resposta B está incorreta porque H também perderá em uma jurisdição
de aviso de corrida. Além disso, Dawn está ocupando o imóvel, o que levantaria uma questão de
notificação de inquérito. No entanto, em uma jurisdição com um estatuto de raça pura, não nos
importamos com o que Hardplace sabia. Contanto que ele registre sua ação primeiro, o que ele fez,
e o que Rock fez, ele deve ganhar. Você pode pensar que Dawn perderia em qualquer jurisdição
porque ela não pagou valor e não gravou, mas lembre-se que nem gravação nem contraprestação
são necessárias para uma transmissão válida; portanto, a resposta D está incorreta. Você pode
pensar que Dawn venceria em qualquer jurisdição por causa do conhecimento de Hardplace e
Rock, mas, novamente, o estatuto de raça pura não considera isso, focando nos benefícios de
certeza de confiar apenas no tempo dos documentos gravados e, portanto, a resposta C está
incorreta.
Leitura recomendada:
Páginas 437-439, 446-451 de Dukeminier, Propriedade, Edição Concisa 2E
3

Oscar presenteou Blackacre com Alice, que não registrou a escritura. Alice então
vendeu Blackacre por valor para Bailey, que imediatamente registrou essa escritura.
Bailey não tomou posse de Blackacre porque morava longe; ela pretendia apenas
mantê-lo para fins de investimento. Ela deixou Oscar continuar morando lá, sem
aluguel, enquanto decidia o que fazer com o imóvel. Vários anos depois, Oscar
precisava de dinheiro, então ele conseguiu um empréstimo do First Bank dando ao
First Bank uma hipoteca sobre Blackacre. O First Bank não sabia das transferências
anteriores. Pouco tempo depois, Oscar deu calote no empréstimo, tendo gasto todo o
dinheiro do empréstimo. O First Bank agora busca fechar a Blackacre. Bailey contesta
alegando que Oscar não era mais dono da Blackacre quando deu a hipoteca ao First
Bank. Em uma jurisdição de notificação que usa um índice concedente-donatário, qual
das seguintes afirmações é verdadeira?

(A) A maioria dos tribunais manteria para o First Bank, porque ele não tinha
conhecimento da transmissão para Bailey.
(B) A maioria dos tribunais manteria Bailey, porque ela gravou primeiro e, portanto, o
First Bank tinha aviso prévio.

(C) A maioria dos tribunais manteria para o First Bank, porque Alice não era
compradora de valores.

(D) A maioria dos tribunais manteria para o First Bank, porque Alice não poderia
transmitir um bom título a Bailey até que ela registrasse sua escritura de Oscar.
Lógica:
A resposta correta é A. Para responder a problemas de ato de gravação, você deve primeiro
considerar quem ganha na ausência de um ato de gravação (Também ajuda a desenhar um
diagrama com setas indicando quem fez o quê e quando.) Neste caso, Oscar transmitiu para Alice,
que então transmitiu para Bailey. Em seguida, Oscar deu uma hipoteca ao First Bank. Na ausência
de um ato de gravação, Bailey vence porque ela foi a primeira na vez. Na época em que Oscar
hipotecou Blackacre, ele não era mais dono dela. Portanto, a única maneira de o First Bank ganhar
é qualificar-se como um comprador de boa-fé (BFP) sob o ato de registro da jurisdição. O problema
diz que esta é uma jurisdição de notificação, o que significa que o First Bank deve ser um
comprador subsequente para o valor que tomou sem aviso da transação anterior. "Comprador"
normalmente inclui hipotecas como o First Bank, então a única questão real é se o banco tinha
aviso. Verifique todos os três tipos de aviso: real, construtivo (registro) e inquérito. O problema diz
que o First Bank não tinha aviso prévio. Além disso, não há razão para pensar que qualquer
circunstância colocaria o First Bank em investigação. O problema diz que Oscar ainda está na
posse. (Se Bailey estivesse na posse, o banco precisaria perguntar por quê.) Portanto, a única
questão é se a gravação de Bailey deu ao banco um aviso de construção. Sua escritura foi
registrada quando o First Bank tomou a hipoteca, mas não de uma forma que o banco pudesse
encontrá-la. Em um índice de outorga, o First Bank procurava o nome de Oscar e não encontrava
nada porque a escritura de Oscar para Alice não estava registrada. A escritura de Alice para Bailey
foi registrada, mas apenas sob os nomes Alice e Bailey; não há razão para o First Bank pesquisar
seus nomes. Portanto, a maioria dos tribunais consideraria que o First Bank era um BFP e,
portanto, pode penhorar a Blackacre a menos que o empréstimo hipotecário seja pago. A resposta
B está incorreta porque, embora a escritura de A a B tenha sido registrada, ela estava fora da
cadeia de títulos e, portanto, não deu aviso prévio. A resposta C está incorreta porque não importa
se Alice (ou Bailey, aliás) pagou valor. Em uma questão de ato de gravação, é apenas o comprador
subsequente que deve pagar o valor. A resposta D está incorreta porque a gravação não é
necessária para transmitir um bom título. A gravação simplesmente dá aviso; não afeta a validade
da transação.
Leitura recomendada:
Páginas 437-439, 446-451 de Dukeminier, Propriedade, Edição Concisa 2E
4

Oscar possuía Blackacre que era adjacente a Whiteacre, de propriedade da Igreja. Os


membros da Igreja usaram parte do Blackacre, chamado Parcel A, como
estacionamento por muitos anos. Oscar decidiu dar a Parcela A para a Igreja, então ele
lhes deu uma escritura de presente para aquela parte de Blackacre. A Igreja, no
entanto, não registrou esse ato. Oscar vendeu Blackacre para Able e contou-lhe sobre
a escritura anterior para Parcel A. Oscar transmitido a Able por escritura de desistência.
Vários meses depois, Able vendeu Blackacre para Baker, pelo valor total de Blackacre,
incluindo Parcel A. Able disse a Baker que poderia haver problemas com relação ao
título da Parcela A, mencionando que "Oscar acha que ele deu à Igreja de alguma
forma". Baker pesquisou o título, no entanto, e não encontrou nada com o que se
preocupar. Able então transmitiu Blackacre a Baker por uma escritura de garantia geral,
que Baker prontamente registrou. Baker então tentou impedir a Igreja de usar a Parcela
A; respondeu mostrando-lhe a escritura de presente de Oscar. Baker agora processa
Able e a Igreja; ou ele quer que a reivindicação da Igreja à Parcela A seja anulada ou
ele quer indenização da Able. Em uma jurisdição de notificação, qual das seguintes
afirmações é verdadeira?

(A) A Baker não tem nenhuma reivindicação contra a Able sob a escritura geral de
garantia, porque a Able lhe informou sobre o potencial problema com a Parcela A.

(B) A Igreja não tem boa titularidade da Parcela A, pois não registrou e não é
adquirente posterior pelo valor.

(C) Baker perderá contra a Igreja, porque ele tinha pelo menos conhecimento da
reivindicação da Igreja.

(D) Baker tem uma reivindicação contra Oscar sob as garantias da escritura, porque
Oscar criou o problema do título.
Lógica:
A resposta correta é C. A primeira questão é se a Igreja tem um bom título para a Parcela A, apesar
de sua falha em registrar. Em uma jurisdição de notificação, Baker tem que ser um comprador
subsequente para o valor que tomou sem aviso prévio. Baker comprou pelo valor, de modo que
parte do teste de PBF foi cumprida. No entanto, ele tinha uma notificação de consulta a respeito do
interesse da Igreja. Mesmo que ele tenha pesquisado o título e não encontrado a escritura, ele
deveria ter perguntado à Igreja com base no comentário de Able, especialmente porque a Igreja
estava em posse. Portanto, a resposta B está incorreta e C está correta. Com relação às garantias
da escritura, Baker deve ser capaz de processar Able sob o pacto de seisin ou o pacto de gozo
tranquilo. A menos que Able tenha excluído especificamente a Parcela A das garantias da escritura,
ele está garantindo um bom título, independentemente do que ele disse a Baker. Portanto, a
resposta A está incorreta. A resposta D também está incorreta porque Oscar não garantiu título; Ele
deu uma escritura de demissão, que não contém garantias.

Capítulo 10
1

Abdul possuía Blackacre, uma área de 40 acres de terra em uma área agrícola,
adjacente a Whiteacre, que é de propriedade de seu bom amigo Nguyen. Por muitos
anos, Abdul permitiu que Nguyen cruzasse Blackacre com seu gado e equipamentos
agrícolas para chegar a outro campo que Nguyen possui do outro lado de Blackacre
chamado Greenacre. Abdul agora está se preparando para vender Blackacre para
Betsy e quer ter certeza de que Nguyen será capaz de continuar a usar o direito de
passagem. Qual das seguintes seria a pior maneira de Abdul conseguir isso?

(A) Abdul poderia redigir e executar uma servidão expressa para Nguyen e registrá-la,
e então escriturar a propriedade para Betsy "sujeito a todos os ônus de registro".
(B) Abdul poderia escriturar Blackacre para Betsy "sujeito a uma servidão em
favor de Nguyen para entrada e saída".

(C) Abdul poderia escriturar Blackacre para Betsy "reservando ao outorgante uma
servidão de entrada e saída entre Whiteacre e Greenacre". Abdul poderia então
escriturar essa servidão a Nguyen.

(D) Abdul poderia dizer a Betsy que só venderia para ela se ela chegasse a algum
acordo com Nguyen sobre o acesso e fizesse disso uma condição de fechamento.
Então, no fechamento, Abdul faria uma escritura de título simples para Betsy e Betsy
então faria uma servidão para Nguyen.
Lógica:
A resposta correta é B. A resposta A é a maneira mais limpa de realizar essa transação. Uma vez
criada uma servidão, trata-se de um ônus sobre Blackacre e Abdul simplesmente indicaria na
escritura que está transmitindo título simples, exceto para esta servidão ou outras servidões de
registro. A resposta B é a mais problemática porque muitas jurisdições aderem à antiga regra da
common law que proíbe a criação de uma servidão em um "estranho à escritura" (ou seja, alguém
que não seja o concedente e o donatário). A regra faz algum sentido; Para criar uma servidão
expressa é preciso entregar a escritura de outorga ao donatário. Aqui, a escritura está sendo
entregue a Betsy, não a Nguyen. Algumas jurisdições abandonaram essa regra porque ela frustra a
clara intenção do concedente, mas não é uma maneira muito boa de fazer isso mesmo em
jurisdições onde ela pode ser permitida porque atrapalha os registros de títulos. O procedimento
estabelecido na resposta C funcionaria. O concedente pode certamente reservar uma servidão para
si mesmo e depois transmiti-la. Não é o melhor caminho, porém, porque causaria confusão sobre
qual propriedade constituía a parcela dominante. A resposta D também funcionaria e poderia até
ser preferível se Betsy quisesse negociar com Nguyen sobre os termos da servidão (escopo,
localização, etc.).
Leitura recomendada:
Páginas 485-492 de Dukeminier, Propriedade, Edição Concisa 2E
2

Cora possui propriedades adjacentes, Blackacre e Whiteacre. Em 2010, ela transmitiu


Blackacre para seu filho, Devin. Ela incluiu uma servidão sobre Whiteacre na escritura
para que Devin pudesse chegar facilmente à estrada principal. Em 2012, Devin vendeu
Blackacre para Edie. Sua escritura transmitindo Blackacre para Edie não mencionou a
servidão sobre Whiteacre e ela não tentou usá-la. Em 2014, Edie levou Blackacre de
volta para Cora. Novamente, não houve menção à servidão na escritura. Em 2016,
Cora transmitiu Blackacre à Fagin, mas não incluiu a servidão na escritura. Fagin quer
usar a servidão que descobriu na busca pelo título. Todos os atos acima foram
devidamente registrados. Cora alega que a servidão não é mais válida. Quem ganha?

(A) Cora, porque a servidão terminou em 2012, quando Devin transmitiu a Edie

(B) Cora, porque a servidão terminou em 2014, devido à fusão


(C) Cora, porque a servidão acabou por abandono

(D) Fagin, porque a servidão é apropriada a Blackacre


Lógica:
A resposta correta é B. A doutrina da fusão sustenta que a servidão se extingue quando as parcelas
dominantes e servientes são de propriedade do mesmo proprietário, porque não se pode ter
servidão sobre o próprio terreno. Portanto, quando Edie voltou para Cora, em 2014, a servidão
acabou. Quando Cora transmitiu para Fagin, ela teria que recriar a servidão, o que ela não fez.
Portanto, a resposta D está incorreta. A resposta A está incorreta porque essa servidão é
apropriada à Blackacre e está intrínseca em seu título; não é pessoal para Devin. Portanto,
permaneceu válido quando Devin transmitiu a Edie, mesmo que ele não o mencionasse na
escritura. A resposta C está incorreta porque, embora Edie não tenha usado a servidão, não há
indícios de que ela pretendia abrir mão de seus direitos a ela. A mera não utilização não configura
abandono.
Leitura recomendada:
Página 532 de Dukeminier, Propriedade, Edição Concisa 2E
3

Em 1990, Albert possuía Whiteacre e Bertil possuía o terreno adjacente, Blackacre.


Alberto concedeu a Bertil uma servidão sobre Whiteacre "com o propósito de minerar
uma caixa de cascalho em Blackacre". A servidão foi devidamente registrada. De 2000
a 2015, Bertil fez uso esporádico da servidão à medida que a caixa de brita
gradualmente se tornou menos rentável. A grama crescia sobre a pista de terra e era
pouco visível em 2010. Em 2010, Albert transmitiu Whiteacre a Candace, mas não
mencionou a servidão, nem oralmente nem na escritura. Em 2017, Bertil não usava a
servidão há muitos anos. Bertil disse a várias pessoas que, uma vez que o preço do
cascalho aumente, ele pode voltar a usar a propriedade para mineração. Em 2017,
encheu o poço de água e começou a alugá-lo a grupos para camping e atividades de
esportes aquáticos. O uso da servidão, portanto, aumentou significativamente.
Candace já processou Bertil para ordenar o uso da servidão. Qual dos seguintes é o
desfecho mais provável deste caso?

(A) O tribunal entenderá que a servidão terminou quando Albert vendeu Whiteacre para
Candace.

(B) O tribunal entenderá que a servidão foi extinta por abandono.

(C) O tribunal determinará o uso da servidão por Bertil para uma nova finalidade.

(D) O tribunal entenderá que a Bertil tem uma servidão de direito de passagem
expressa que pode ser usada para esse fim, que é uma "evolução natural" do uso
anterior.
Lógica:
A resposta correta é C. A servidão ainda existe, mas tem alcance limitado. A resposta A está
incorreta porque a servidão é um ônus para a Blackacre e permanece em vigor enquanto o
comprador subsequente tiver aviso. Foi gravado aqui, então permaneceu um estorvo quando
Candace tomou posse. O abandono é uma questão mais próxima; embora este seja um longo
período de desuso, o tribunal deve ter alguma indicação, expressa ou implícita, de que Bertil
pretendia abrir mão de seus direitos. Um tribunal pode considerar que o uso da propriedade para
uso recreativo pode ser um indício de que Bertil pretendia abandonar o uso da mineração de
cascalho, mas há evidências de que isso foi uma interrupção temporária. (Ele disse às pessoas que
poderia começar a minerar novamente quando o preço do cascalho subisse.) Assim, embora a
servidão provavelmente ainda exista, o uso atual está além do escopo da servidão. A linguagem
limita claramente o uso "para mineração de cascalho". Embora alguma "evolução natural" de uma
servidão seja permitida, o uso recreativo é completamente diferente da mineração e apresenta
diferentes encargos sobre a propriedade serviente. Portanto, um tribunal provavelmente decretaria
o novo uso de Bertil, mas sustentaria que a servidão para mineração ainda existe.
Leitura recomendada:
Páginas 515-523 de Dukeminier, Propriedade, Edição Concisa 2E
4

Em 2012, Kevin comprou o lote 1 do loteamento Greenacres na cidade de Anytown.


Quando o loteamento foi originalmente emplacado, em 1927, a incorporadora fez um
convênio em todos os lotes, restringindo-os apenas ao uso residencial. Em 1955,
Anytown zoneou a propriedade R-1, que permite apenas o uso residencial unifamiliar. A
rua em que fica a propriedade de Kevin, antes uma estrada de duas pistas, agora se
tornou uma grande via, e vários negócios e uma grande igreja agora existem do outro
lado da rua do loteamento. Uma moradora de uma rua do interior usa sua casa para
uma pequena creche, e outra moradora é uma contadora que recebe clientes em sua
casa. Kevin gostaria de usar sua casa no Lote 1 para um escritório de advocacia que
ele espera que tenha pouco trânsito e nenhum barulho. Ele pode provar que a casa
vale US $ 200.000 como residência, mas cerca de US $ 300.000 se o uso comercial
como este for permitido. Seus vizinhos no loteamento Greenacres se opõem, no
entanto, e dizem que pretendem fazer cumprir o pacto contra seu uso proposto. Se
você estivesse representando Kevin, qual dos seguintes seria seu melhor argumento
em relação à exigibilidade do pacto?

(A) O pacto que restringe a propriedade de Kevin deve ser considerado inexequível
devido a "circunstâncias alteradas".

(B) O pacto não pode ser executado contra Kevin, porque não há privação horizontal.

(C) O pacto deve ser considerado renunciado.

(D) O pacto não pode ser cumprido contra Kevin, porque os vizinhos não podem mais
provar um esquema comum de desenvolvimento.
Lógica:
A resposta correta é C. A resposta C é o melhor argumento, pois a existência da creche e do
escritório do contador pode ser prova de que o convênio foi dispensado. A renúncia pode ser
argumentada com sucesso se violações anteriores de um pacto forem semelhantes à violação
proposta em relação aos danos que causam. Aqui, o uso proposto por Kevin envolveria apenas um
pequeno aumento no tráfego, provavelmente não maior do que o causado pela operação da creche
e do escritório do contador, e nenhum outro dano aos outros proprietários de imóveis no
loteamento. A resposta A exige a prova de "circunstâncias alteradas", o que é um fardo difícil de
cumprir. Os tribunais normalmente exigem que as mudanças tenham frustrado o propósito do pacto,
o que não parece ser o caso aqui. Além disso, as mudanças precisam ocorrer dentro do loteamento
e não fora dele, o que aconteceu aqui. Então, A é um argumento possível, mas não ótimo. A
resposta B não é inicial; havia privação horizontal (a variedade concedente-donatário). A resposta D
também está incorreta; o argumento do esquema comum é usado para implicar um pacto em lotes
que não estão sujeitos a um pacto explícito, como o lote de Kevin.
Leitura recomendada:
Páginas 545-551 de Dukeminier, Propriedade, Edição Concisa 2E
5

Mia possuía Blackacre que era limitado a leste por Whiteacre, de propriedade de
Nelson. Ambas as propriedades eram delimitadas pela River Road ao sul e pelo Big
River ao norte. Um dia, Nelson disse a Mia que estava planejando construir uma nova
casa mais atrás em Whiteacre, em um cume com uma boa vista do rio abaixo.
"Infelizmente", disse ele, "não há uma boa maneira de voltar lá na minha propriedade
por causa de um grande barranco e pântano. Você se importaria se eu passasse por
Blackacre para voltar lá?" "Claro, posso deixar você fazer isso, Nelson", respondeu
Mia. A construtora que Nelson contratou construiu uma estrada de terra no extremo
leste de Blackacre e a usou para chegar ao canteiro de obras, onde construiu uma bela
casa. Nelson mudou-se e continuou a usar a estrada de terra sobre Blackacre pelos
oito anos seguintes. Nelson então morreu e sua filha Olivia herdou Whiteacre. Ela usou
a estrada também por mais um ano. Mia então vendeu Blackacre para Portia. Portia
logo se cansou de ter um estranho passando por cima de sua propriedade, então ela
bloqueou o acesso de Olivia. Portia afirma que Nelson tinha, no máximo, uma licença
pessoal revogável. Olívia quer entrar com uma ação para estabelecer seu direito à
servidão. O prazo prescricional para prescrição é de sete anos nessa jurisdição. Qual
das seguintes teorias teria maior probabilidade de ajudar Olivia neste caso?

(A) Servidão por prescrição

(B) Servidão por preclusão

(C) Servidão implícita pela necessidade

(D) Servidão implícita pelo uso prévio


Lógica:
A resposta correta é B. A servidão por preclusão se encaixa nos fatos: exige confiança razoável e
previsível do proprietário da parcela dominante (Nelson) em uma promessa do proprietário da
parcela serviente (Mia). Mia prometeu a Nelson que poderia usar a estrada, e ele confiou nessa
promessa para construir uma casa cara. Portanto, a licença pode ter amadurecido em uma servidão
irrevogável de tal forma que Portia agora está vinculada a ela. Embora o problema não pergunte,
Portia estaria amarrada se ela tomasse conhecimento, o que provavelmente seria aviso de inquérito
aqui. Além disso, como a servidão era apropriada a Whiteacre, Olivia poderia continuar a usá-la.
Uma servidão por prescrição, resposta A, não funcionará porque o uso foi permissivo. Uma
servidão implícita por necessidade ou implícita por uso prévio também não funcionará porque não
havia propriedade comum das duas parcelas. Essas teorias dependem de uma servidão ser
implícita como "parte do negócio" quando a propriedade é dividida por um proprietário comum. Isso
não aconteceu aqui. Portanto, as respostas C e D estão incorretas. Uma servidão legal por
necessidade pode ser uma possibilidade, mas essa não é uma escolha listada.
Leitura recomendada:
Páginas 492-496 de Dukeminier, Propriedade, Edição Concisa 2E
6

Olivia possuía uma faixa de terra limitada ao norte pela Green Road e ao sul pela
Brown Road. Em 1996, Olivia construiu uma casa na metade norte da propriedade que
ela chamou de Greenacre. Para chegar à casa, ela às vezes usava a entrada da frente
que levava ao norte para a Green Road, mas depois de fortes nevascas ou fortes
chuvas, a Green Road era acessível apenas com um veículo de tração nas quatro
rodas. Por isso, ela costumava usar "o caminho de volta" - uma trilha de terra que
passava pela metade sul da propriedade, que ela chamava de Brownacre, levando à
Brown Road. Em 2006, Olivia vendeu Greenacre para Andrew e ela construiu e se
mudou para uma casa em Brownacre. Embora a escritura não dissesse nada sobre
uma servidão sobre Brownacre, Andrew frequentemente usava a trilha de terra para
chegar à Brown Road, que era uma maneira mais rápida de chegar ao seu local de
trabalho. Olívia muitas vezes o via e acenava olá. Em 2014, um deslizamento de terra
fechou a Green Road, e as autoridades do condado notificaram Andrew de que os
danos na estrada eram graves e poderiam não ser reparados por anos. Depois disso,
Andrew usou exclusivamente a pista de terra sobre Brownacre para entrada e saída.
Em 2016, Olivia vendeu Brownacre para Betsy. Logo depois, Betsy colocou um portão
trancado na pista de terra e proibiu Andrew de usá-lo. Nessa jurisdição, o prazo
prescricional para prescrição é de 10 anos. Qual das seguintes é a melhor teoria de
Andrew para uma servidão sobre Brownacre?

(A) Estoppel

(B) Implícito por necessidade

(C) Prescrição

(D) Implícito pelo uso prévio

Lógica:
A resposta correta é D. Uma servidão implícita pelo uso prévio requer um proprietário comum
(Olivia); uso prévio (ela usava a pista de terra para a própria Brown Road com frequência);
necessidade razoável (mais fácil de atender do que necessidade estrita e provavelmente atendida
aqui devido a dificuldades no inverno), e aviso (a pista de terra seria visível). Justificar-se-ia um
comprador razoável em presumir que o uso da pista de terra foi incluído no negócio? Talvez – este
não é um slam dunk, mas é a melhor chance para Andrew. A outra possibilidade é uma opção que
não foi listada: uma servidão estatutária por necessidade, que exigiria que Andrew compensasse
Betsy pelo uso da pista e dependeria dos requisitos legais na jurisdição em que o terreno está
localizado. A resposta A está incorreta porque uma servidão por preclusão requer confiança
razoável em uma promessa feita por Olivia (ou Betsy); aqui não havia essa promessa. Você pode
pensar que uma servidão implícita por necessidade seria possível, resposta B, dado que Andrew
não tem nenhuma maneira real de sair de sua propriedade a não ser através de Brownacre. No
entanto, uma servidão implícita por necessidade exige que a necessidade surja no momento em
que o imóvel foi dividido em dois pelo proprietário comum. Em 2010, quando Olivia vendeu a
Greenacre para Andrew, ele pôde usar a Green Road. Os fatos indicam que às vezes era difícil –
exigia um veículo com tração nas quatro rodas –, mas os tribunais geralmente exigem estrita
necessidade, o que esses fatos não suportam. A prescrição, resposta C, também não adianta; O
uso adverso provavelmente só começou em 2016. Antes disso, o uso provavelmente era permissivo
porque Olivia viu Andrew usando a pista e parecia não ter nenhum problema com ela.
Leitura recomendada:
Páginas 497-504 de Dukeminier, Propriedade, Edição Concisa 2E
7

Olivia possuía uma faixa de terra limitada ao norte pela Green Road e ao sul pela
Brown Road. Havia uma casa na metade norte da propriedade, que Olivia chamou de
Greenacre, com uma entrada que levava à Green Road, e uma trilha de terra, sobre a
metade sul da propriedade, chamada Brownacre, que levava à Brown Road. Em 2006,
Olivia vendeu Greenacre para Andrew. Embora a escritura não dissesse nada sobre
uma servidão sobre Brownacre, Andrew frequentemente usava a pista de terra. Depois
que um deslizamento de terra fechou a Green Road em 2014, que permaneceu
fechada por anos, Andrew usou exclusivamente a pista de terra sobre Brownacre.
Olivia então vendeu Brownacre para Betsy em 2016, que instalou uma cerca em toda a
pista de terra para impedir Andrew de usá-la. Nessa jurisdição, o prazo prescricional
para prescrição é de 10 anos. Andrew processou para poder continuar a usar a pista de
terra, e um tribunal determinou que Andrew tinha uma servidão legal sobre Brownacre.
O condado finalmente reparou e reabriu a Green Road, e Andrew parou de usar a pista
de terra. Vários meses depois, Betsy perguntou se ele pretendia usar a faixa
novamente, e Andrew respondeu "Não". Dois anos depois, outro deslizamento de terra
fechou a Estrada Verde, e Andrew queria usar a servidão novamente. Qual dos
seguintes seria o melhor argumento de Betsy de que a servidão terminou?

a) Abandono

(B) Estoppel

(C) Liberação

(D) Prescrição
Lógica:
A resposta correta é A. O abandono exige um período de não utilização acompanhado de uma
intenção de abandono. Aqui, Andrew parou de usá-lo por um período considerável e indicou sua
intenção de abandoná-lo dizendo que estava desistindo dele. Não foi um lançamento porque um
lançamento deve ser por escrito e assinado por Andrew para satisfazer o Estatuto de Fraudes,
tornando a resposta C incorreta. Estoppel poderia encerrar a servidão se Betsy tivesse mudado sua
posição na promessa de Andrew de parar de usá-la, mas não há indicação de que ela tenha feito
algo em seu prejuízo. Assim, a resposta B está incorreta. Finalmente, uma servidão pode ser
extinta por prescrição, mas isso exigiria que Betsy bloqueasse o uso da servidão por Andrew pelo
período prescritivo que sabemos ser de dez anos. Isso não aconteceu aqui. Portanto, a resposta D
está incorreta.
Leitura recomendada:
Páginas 523-533 de Dukeminier, Propriedade, Edição Concisa 2E
8

pediu à cidade de Maryville para rezonear uma área de terra de R-1 (residencial
unifamiliar) para R-3 (residencial multifamiliar), para que pudesse limpar algumas
moradias bastante precárias e construir alguns novos apartamentos e condomínios. No
entanto, a prefeitura estava preocupada em deslocar um grande número de moradores
de baixa renda. Por isso, a Prefeitura permitiu o rezoneamento em troca de um pacto
restritivo que exige que pelo menos 25% das unidades habitacionais disponíveis no
novo projeto sejam reservadas para moradores de baixa renda. A Urban Development
teve problemas para financiar o projeto e, depois de dois anos, vendeu todo o local
para a High End Housing, Inc., que não tem intenção de cumprir o convênio. Qual dos
seguintes é menos provável que seja um possível problema com a aplicação do pacto
na lei contra High End?

(A) O ônus do pacto não pode correr com o terreno para os proprietários posteriores do
empreendimento, pois o benefício é bruto.

(B) O ônus do pacto não pode recair sobre os proprietários posteriores do


empreendimento, pois não pode "tocar e preocupar" o terreno.

(C) O ônus do convênio não pode recair sobre os proprietários posteriores do


empreendimento, pois não há privação horizontal entre o Município e a Urbanismo, Inc.

(D) O pacto pode não ser exequível porque não há privação entre o Município e o
Município.

Lógica:
A resposta correta é D. A privação horizontal é exigida (entre o Desenvolvimento Urbano e a
Cidade), e a prividade vertical é exigida (entre o Desenvolvimento Urbano e o High End), mas a
prividade não é exigida entre a Cidade e o High End. A resposta A está incorreta porque algumas
jurisdições ainda aplicam a antiga regra inglesa contra permitir que o ônus de um pacto corra onde
o benefício é mantido em bruto. Esses tribunais, e alguns estatutos, exigem que haja um benefício
direto para algum outro imóvel, o que não há aqui. Os comentaristas normalmente denunciam a
regra como anacrônica, mas diz-se que ela persiste. Ver Thomas E. Roberts, "Promises Respect
Land Use — Can Benefits Be Held in Gross", p. 51 Mo. L. Rev. (1986). A resposta B levanta a
questão do que entendemos por requisito "toque e preocupação". Pode-se dizer que esse pacto
toca e diz respeito à terra, porque certamente tem a ver com a forma como a terra é usada. No
entanto, também se pode argumentar que isso não mudará realmente o que é construído no terreno
ou como os apartamentos parecem ou quantas pessoas vivem lá, mas apenas mudará as práticas
comerciais dos proprietários, exigindo que eles tenham uma qualificação de renda para 25% dos
moradores. Os tribunais poderiam justificar qualquer uma das conclusões, tornando-a, portanto,
pelo menos um possível argumento para High End aqui. A resposta C também é um possível
argumento para o High End porque, sob as regras tradicionais, a privação horizontal não existe
entre a Cidade e o Desenvolvimento Urbano. Se essa jurisdição ainda adere ao requisito de
privação horizontal, o tribunal deve considerar que não há privação concedente-donatário, mútuo ou
locador-inquilino neste caso.

Você também pode gostar